Exam 5

Pataasin ang iyong marka sa homework at exams ngayon gamit ang Quizwiz!

13. The sister of a patient with AD asks the nurse whether prevention of the disease is possible. In responding, the nurse explains that there is no known way to prevent AD but there are ways to keep the brain healthy. What is included in the ways to keep the brain healthy (select all that apply)? a. Avoid trauma to the brain. b. Recognize and treat depression early. c. Avoid social gatherings to avoid infections. d. Do not overtax the brain by trying to learn new skills. e. Daily wine intake will increase circulation to the brain. f. Exercise regularly to decrease the risk for cognitive decline

13. a, b, f. Avoiding trauma to the brain, treating depression early, and exercising regularly can maintain cognitive function. Staying socially active, avoiding intake of harmful substances, and challenging the brain to keep its connections active and create new ones also help to keep the brain healthy.

10. The patient is receiving donepezil (Aricept), lorazepam (Ativan), risperidone (Risperdal), and sertraline (Zoloft) for the management of AD. What benzodiazepine medication is being used to help manage this patient's behavior? a. Sertraline (Zoloft) b. Donepezil (Aricept) c. Lorazepam (Ativan) d. Risperidone (Risperdal)

10. c. Lorazepam (Ativan) is a benzodiazepine used to manage behavior with AD. Sertraline (Zoloft) is a selective serotonin reuptake inhibitor used to treat depression. Donepzil (Aricept) is a cholinesterase inhibitor used for decreased memory and cognition. Risperidone (Risperdal) is an antipsychotic used for behavior management.

11. What N-methyl-d-aspartate (NMDA) receptor antagonist is frequently used for a patient with AD who is experiencing decreased memory and cognition? a. Trazodone (Desyrel) b. Olanzapine (Zyprexa) c. Rivastigmine (Exelon) d. Memantine (Namenda)

11. d. Memantine (Namenda) is the N-methyl-d-aspartate (NMDA) receptor antagonist frequently used for AD patients with decreased memory and cognition. Trazodone (Desyrel) is an atypical antidepressant that may help with sleep problems. Olanzapine (Zyprexa) is an antipsychotic medication used for behavior management. Rivastigmine (Exelon) is a cholinesterase inhibitor used for decreased memory and cognition.

12. A patient with AD in a long-term care facility is wandering the halls very agitated, asking for her "mommy" and crying. What is the best response by the nurse? a. Ask the patient, "Why are you behaving this way?" b. Tell the patient, "Let's go get a snack in the kitchen." c. Ask the patient, "Wouldn't you like to lie down now?" d. Tell the patient, "Just take some deep breaths and calm down."

12. b. Patients with moderate to severe AD frequently become agitated but because their short-term memory loss is so pronounced, distraction is a very good way to calm them. "Why" questions are upsetting to them because they don't know the answer and they cannot respond to normal relaxation techniques.

A patient has ICP monitoring with an intraventricular catheter. A priority nursing intervention for the patient is a. aseptic technique to prevent infection b. constant monitoring of ICP waveforms c. removal of CSF to maintain normal ICP d. sampling CSF to determine abnormalities

A. Aseptic technique to prevent infection- An intraventricular catheter is a fluid coupled system that can provide direct access for microorganisms to enter the ventricles of the brain, and aseptic technique is a very high nursing priority to decrease the risk for infection. Constant monitoring of ICP waveforms is not usually necessary, and removal of CSF for sampling or to maintain normal ICP is done only when specifically ordered

The nurse is monitoring a patient for increased ICP following a head injury. Which of the following manifestations indicate an increased ICP (select all that apply) a. fever b. oriented to name only c. narrowing pulse pressure d. dilated right pupil > left pupil e. decorticate posturing to painful stimulus

A, B, D, E The first sign of increased ICP is a change in LOC. Other manifestations are dilated ipsilateral pupil, changes in motor response such as posturing, and fever, which may indicate pressure on the hypothalamus. Changes in vital signs would be an increased systolic BP with widened pulse pressure and bradycardia.

The nurse is caring for a patient admitted for evaluation and surgical removal of a brain tumor. The nurse will plan interventions for this patient based on knowledge that brain tumors can lead to which complications (select all that apply)? A. Vision loss B. Cerebral edema C. Pituitary dysfunction D. Parathyroid dysfunction E. Focal neurologic deficits

A,B,C,E Brain tumors can manifest themselves in a wide variety of symptoms depending on location, including vision loss and focal neurologic deficits. Tumors that put pressure on the pituitary can lead to dysfunction of the gland. As the tumor grows, clinical manifestations of increased intracranial pressure (ICP) and cerebral edema appear. The parathyroid gland is not regulated by the cerebral cortex or the pituitary gland.

A patient has a nursing diagnosis of risk for ineffective cerebral tissue perfusion related to cerebral edema. An appropriate nursing intervention for the patient is a. avoiding positioning the patient with neck and hip flexion b. maintaining hyperventilation to a PaCO2 of 15 to 20 mm Hg c. clustering nursing activities to provide periods of uninterrupted rest d. routine suctioning to prevent accumulation of respiratory secretions

A. Nursing care activities that increase ICP include hip and neck flexion, suctioning, clustering care activities, and noxious stimuli; they should be avoided or performed as little as possible in the patient with increased ICP. Lowering the PaCO2 below 20 mm Hg can cause ischemia and worsening of ICP; the PaCO2 should be maintained at 30 to 35 mm Hg.

The nurse is discharging a patient admitted with a transient ischemic attack (TIA). For which medications might the nurse expect to provide discharge instructions (select all that apply)? A. Clopidogrel (Plavix) B. Enoxaparin (Lovenox) C. Dipyridamole (Persantine) D. Enteric-coated aspirin (Ecotrin) E. Tissue plasminogen activator (tPA)

A. Clopidogrel (Plavix) C. Dipyridamole (Persantine) D. Enteric-coated aspirin (Ecotrin) Aspirin is the most frequently used antiplatelet agent. Other drugs to prevent clot formation include clopidogrel (Plavix), dipyridamole (Persantine), ticlopidine (Ticlid), combined dipyridamole and aspirin (Aggrenox), and anticoagulant drugs, such as oral warfarin (Coumadin). Tissue plasminogen activator is a fibrinolytic medication used to treat ischemic stroke not prevent TIAs or strokes.

Which modifiable risk factors for stroke would be most important for the nurse to include when planning a community education program? A. Hypertension B. Hyerlipidemia C. Alcohol consumption D. Oral contraceptive use

A. Hypertension Hypertension is the single most important modifiable risk factor, but it is still often undetected and inadequately treated. The public is often more aware of hyperlipidemia and oral contraceptive use as risk factors for stroke. Alcohol is also a modifiable risk factor.

Computed tomography (CT) of a 68-year-old male patient's head reveals that he has experienced a hemorrhagic stroke. What is the priority nursing intervention in the emergency department? A. Maintenance of patient's airway. B. Positioning to promote cerebral perfusion. C. Control of fluid and electrolyte imbalances. D. Administration of tissue plasminogen activator (tPA)

A. Maintenance of patient's airway. Maintenance of a patent airway is the priority in the acute care of a patient with a hemorrhagic stroke. It supersedes the importance of fluid and electrolyte imbalance and positioning. tPA is contraindicated in hemorrhagic stroke.

The patient with diabetes mellitus has had a right-sided stroke. Which nursing intervention should the nurse plan to provide for this patient related to expected manifestations of this stroke? A. Safety measures B. Patience with communication C. Mobility assistance on the right side D. Place food in the left side of patient's mouth

A. Safety measures A patient with a right-sided stroke has spatial-perceptual deficits, tends to minimize problems, has a short attention span, is impulsive, and may have impaired judgment. Safety is the biggest concern for this patient. Hemiplegia occurs on the left side of this patient's body. The patient with a left-sided stroke has hemiplegia on the right, is more likely to have communication problems, and needs mobility assistance on the right side with food placed on the left side if the patient needs to be fed after a swallow evaluation has taken place.

A patient who has bacterial meningitis is disoriented and anxious. Which nursing action will be included in the plan of care? a. Encourage family members to remain at the bedside. b. Apply soft restraints to protect the patient from injury. c. Keep the room well-lighted to improve patient orientation. d. Minimize contact with the patient to decrease sensory input.

ANS: A Patients with meningitis and disorientation will be calmed by the presence of someone familiar at the bedside. Restraints should be avoided because they increase agitation and anxiety. The patient requires frequent assessment for complications; the use of touch and a soothing voice will decrease anxiety for most patients. The patient will have photophobia, so the light should be dim.

The charge nurse observes an inexperienced staff nurse who is caring for a patient who has had a craniotomy for a brain tumor. Which action by the inexperienced nurse requires the charge nurse to intervene? a. The staff nurse suctions the patient every 2 hours. b. The staff nurse assesses neurologic status every hour. c. The staff nurse elevates the head of the bed to 30 degrees. d. The staff nurse administers a mild analgesic before turning the patient.

ANS: A Suctioning increases intracranial pressure and is done only when the patient's respiratory condition indicates it is needed. The other actions by the staff nurse are appropriate.

When admitting a patient who has a tumor of the right frontal lobe, the nurse would expect to find a. judgment changes. b. expressive aphasia. c. right-sided weakness. d. difficulty swallowing.

ANS: A The frontal lobes control intellectual activities such as judgment. Speech is controlled in the parietal lobe. Weakness and hemiplegia occur on the contralateral side from the tumor. Swallowing is controlled by the brainstem

The nurse obtains these assessment findings for a patient who has a head injury. Which finding should be reported rapidly to the health care provider? a. Urine output of 800 mL in the last hour b. Intracranial pressure of 16 mm Hg when patient is turned c. Ventriculostomy drains 10 mL of cerebrospinal fluid per hour d. LICOX brain tissue oxygenation catheter shows PbtO2 of 38 mm Hg

ANS: A The high urine output indicates that diabetes insipidus may be developing and interventions to prevent dehydration need to be rapidly implemented. The other data do not indicate a need for any change in therapy.

A 68-year-old patient who is hospitalized with pneumonia is disoriented and confused 3 days after admission. Which information indicates that the patient is experiencing delirium rather than dementia? a. The patient was oriented and alert when admitted. b. The patient's speech is fragmented and incoherent. c. The patient is oriented to person but disoriented to place and time. d. The patient has a history of increasing confusion over several years.

ANS: A The onset of delirium occurs acutely. The degree of disorientation does not differentiate between delirium and dementia. Increasing confusion for several years is consistent with dementia. Fragmented and incoherent speech may occur with either delirium or dementia.

To assess the functioning of the trigeminal and facial nerves (CN V and VII), the nurse should a. apply a cotton wisp strand to the cornea. b. have the patient read a magazine or book. c. shine a bright light into the patient's pupil. d. check for unilateral drooping of the eyelids

ANS: A The trigeminal and facial nerves are responsible for the corneal reflex. The optic nerve is tested by having the patient read a Snellen chart or a newspaper. Assessment of pupil response to light and ptosis are used to check function of the oculomotor nerve.

A 68-year-old patient who is hospitalized with pneumonia is disoriented and confused 3 days after admission. Which information indicates that the patient is experiencing delirium rather than dementia? a. The patient was oriented and alert when admitted. b. The patient's speech is fragmented and incoherent. c. The patient is oriented to person but disoriented to place and time. d. The patient has a history of increasing confusion over several years.

ANS: A-The patient was oriented and alert when admitted The onset of delirium occurs acutely. The degree of disorientation does not differentiate between delirium and dementia. Increasing confusion for several years is consistent with dementia. Fragmented and incoherent speech may occur with either delirium or dementia.

A patient who has a head injury is diagnosed with a concussion. Which action will the nurse plan to take? a. Coordinate the transfer of the patient to the operating room. b. Provide discharge instructions about monitoring neurologic status. c. Transport the patient to radiology for magnetic resonance imaging (MRI) of the brain. d. Arrange to admit the patient to the neurologic unit for observation for 24 hours.

ANS: B A patient with a minor head trauma is usually discharged with instructions about neurologic monitoring and the need to return if neurologic status deteriorates. MRI, hospital admission, or surgery are not indicated in a patient with a concussion.

Which problem can the nurse expect for a patient who has a positive Romberg test result? a. Pain c. Aphasia b. Falls d. Confusion

ANS: B A positive Romberg test result indicates that the patient has difficulty maintaining balance when standing with the eyes closed. The Romberg does not test for orientation, thermoregulation, or discomfort.

The nurse is administering a mental status examination to a 48-year-old patient who has hypertension. The nurse suspects depression when the patient responds to the nurse's questions with a. "Is that right?" b. "I don't know." c. "Wait, let me think about that." d. "Who are those people over there?"

ANS: B Answers such as "I don't know" are more typical of depression than dementia. The response "Who are those people over there?" is more typical of the distraction seen in a patient with delirium. The remaining two answers are more typical of a patient with mild to moderate dementia.

After reviewing a patient's cerebrospinal fluid analysis, which result will be most important for the nurse to communicate to the health care provider? a. Specific gravity 1.007 b. Protein 65 mg/dL (0.30 g/L) c. White blood cell (WBC) count 4/μL d. Glucose 45 mg/dL (1.7 mmol/L)

ANS: B The protein level is high. 15 to 60 mg/100 mL. The Specific Gravity 1.006-1.007 WBCs 0 to 5 glucose 50 to 80 mg/100 mL

A patient who has severe Alzheimer's disease (AD) is being admitted to the hospital for surgery. Which intervention will the nurse include in the plan of care? a. Encourage the patient to discuss events from the past. b. Maintain a consistent daily routine for the patient's care. c. Reorient the patient to the date and time every 2 to 3 hours. d. Provide the patient with current newspapers and magazines.

ANS: B Providing a consistent routine will decrease anxiety and confusion for the patient.

Which nursing action will be most effective in ensuring daily medication compliance for a patient with mild dementia? a. Setting the medications up monthly in a medication box b. Having the patient's family member administer the medication c. Posting reminders to take the medications in the patient's house d. Calling the patient weekly with a reminder to take the medication

ANS: B-Having the patient's family member administer the medication Because the patient with mild dementia will have difficulty with learning new skills and forgetfulness, the most appropriate nursing action is to have someone else administer the drug.

Which intervention will the nurse include in the plan of care for a patient with moderate dementia who had an appendectomy 2 days ago? a. Provide complete personal hygiene care for the patient. b. Remind the patient frequently about being in the hospital. c. Reposition the patient frequently to avoid skin breakdown. d. Place suction at the bedside to decrease the risk for aspiration.

ANS: B-Remind the patient frequently about being in the hospital. The patient with moderate dementia will have problems with short- and long-term memory and will need reminding about the hospitalization. The other interventions would be used for a patient with severe dementia, who would have difficulty with swallowing, self-care, and immobility.

A 56-year-old patient in the outpatient clinic is diagnosed with mild cognitive impairment (MCI).Which action will the nurse include in the plan of care? a. Suggest a move into an assisted living facility. b. Schedule the patient for more frequent appointments. c. Ask family members to supervise the patient's daily activities. d. Discuss the preventive use of acetylcholinesterase medications.

ANS: B-Schedule the patient for more frequent appointments

When administering a mental status examination to a patient with delirium, the nurse should a. wait until the patient is well-rested. b. administer an anxiolytic medication. c. choose a place without distracting stimuli. d. reorient the patient during the examination.

ANS: C Because overstimulation by environmental factors can distract the patient from the task of answering the nurse's questions, these stimuli should be avoided. The nurse will not wait to give the examination because action to correct the delirium should occur as soon as possible. Reorienting the patient is not appropriate during the examination. Antianxiety medications may increase the patient's delirium.

When administering the Mini-Cog exam to a patient with possible Alzheimer's disease, which action will the nurse take? a. Check the patient's orientation to time and date. b. Obtain a list of the patient's prescribed medications. c. Ask the person to use a clock drawing to indicate a specific time. d. Determine the patient's ability to recognize a common object such as a pen.

ANS: C In the Mini-Cog, patients illustrate a specific time stated by the examiner by drawing the time on a clock face. The other actions may be included in assessment for Alzheimer's disease, but are not part of the Mini-Cog exam.

The community health nurse is developing a program to decrease the incidence of meningitis in adolescents and young adults. Which nursing action is most important? a. Vaccinate 11- and 12-year-old children against Haemophilus influenzae. b. Emphasize the importance of hand washing to prevent spread of infection. c. Immunize adolescents and college freshman against Neisseria meningitides. d. Encourage adolescents and young adults to avoid crowded areas in the winter.

ANS: C The Neisseria meningitides vaccination is recommended for children ages 11 and 12, unvaccinated teens entering high school, and college freshmen. Hand washing may help decrease the spread of bacteria, but it is not as effective as immunization. Vaccination with Haemophilus influenzae is for infants and toddlers. Because adolescents and young adults are in school or the workplace, avoiding crowds is not realistic.

Which information about a patient who is hospitalized after a traumatic brain injury requires the most rapid action by the nurse? a. Intracranial pressure of 15 mm Hg b. Cerebrospinal fluid (CSF) drainage of 15 mL/hour c. Pressure of oxygen in brain tissue (PbtO2) is 14 mm Hg d. Cardiac monitor shows sinus tachycardia, with a heart rate of 126 beats/min

ANS: C The PbtO2 should be 20 to 40 mm Hg. Lower levels indicate brain ischemia. An intracranial pressure (ICP) of 15 mm Hg is at the upper limit of normal. CSF is produced at a rate of 20 to 30 mL/hour. The reason for the sinus tachycardia should be investigated, but the elevated heart rate is not as concerning as the decrease in PbtO2.

A patient is being evaluated for Alzheimer's disease (AD). The nurse explains to the patient's adult children that a. the most important risk factor for AD is a family history of the disorder. b. new drugs have been shown to reverse AD dramatically in some patients. c. a diagnosis of AD is made only after other causes of dementia are ruled out. d. the presence of brain atrophy detected by magnetic resonance imaging (MRI) will confirm the diagnosis of AD.

ANS: C The diagnosis of AD is usually one of exclusion. Age is the most important risk factor for development of AD

A patient is being evaluated for Alzheimer's disease (AD). The nurse explains to the patient's adult children that a. the most important risk factor for AD is a family history of the disorder. b. new drugs have been shown to reverse AD dramatically in some patients. c. a diagnosis of AD is made only after other causes of dementia are ruled out. d. the presence of brain atrophy detected by magnetic resonance imaging (MRI) will confirm the diagnosis of AD.

ANS: C The diagnosis of AD is usually one of exclusion. Age is the most important risk factor for development of AD. Drugs may slow the deterioration but do not reverse the effects of AD. Brain atrophy is a common finding in AD, but it can occur in other diseases as well and does not confirm a diagnosis of AD.

The care plan for a patient who has increased intracranial pressure and a ventriculostomy includes the following nursing actions. Which action can the nurse delegate to nursing assistive personnel (NAP) who regularly work in the intensive care unit? a. Monitor cerebrospinal fluid color hourly. b. Document intracranial pressure every hour. c. Turn and reposition the patient every 2 hours. d. Check capillary blood glucose level every 6 hours.

ANS: D Experienced NAP can obtain capillary blood glucose levels when they have been trained and evaluated in the skill. Monitoring and documentation of cerebrospinal fluid (CSF) color and intracranial pressure (ICP) require RN-level education and scope of practice. Although repositioning patients is frequently delegated to NAP, repositioning a patient with a ventriculostomy is complex and should be done by the RN.

During the neurologic assessment, the patient cooperates with the nurse's directions to grip with the hands and to move the feet but is unable to respond orally to the nurse's questions. The nurse will suspect a. a brainstem lesion. b. a temporal lobe lesion. c. injury to the cerebellum. d. damage to the frontal lobe.

ANS: D Expressive speech (ability to express the self in language) is controlled by Broca's area in the frontal lobe. The temporal lobe contains Wernicke's area, which is responsible for receptive speech (ability to understand language input). The cerebellum and brainstem do not affect higher cognitive functions such as speech.

Which assessment finding in a patient who was admitted the previous day with a basilar skull fracture is most important to report to the health care provider? a. Bruising under both eyes b. Complaint of severe headache c. Large ecchymosis behind one ear d. Temperature of 101.5° F (38.6° C)

ANS: D Patients who have basilar skull fractures are at risk for meningitis, so the elevated temperature should be reported to the health care provider. The other findings are typical of a patient with a basilar skull fracture.

After having a craniectomy and left anterior fossae incision, a patient has a nursing diagnosis of impaired physical mobility related to decreased level of consciousness and weakness. An appropriate nursing intervention is to a. position the bed flat and log roll the patient. b. cluster nursing activities to allow longer rest periods. c. turn and reposition the patient side to side every 2 hours. d. perform range-of-motion (ROM) exercises every 4 hours.

ANS: D ROM exercises will help to prevent the complications of immobility. Patients with anterior craniotomies are positioned with the head elevated. The patient with a craniectomy should not be turned to the operative side. When the patient is weak, clustering nursing activities may lead to more fatigue and weakness.

A patient is hospitalized with a possible seizure disorder. To determine the cause of the patient's symptoms, the nurse will anticipate the need to teach the patient about which of these tests? a. Cerebral angiography b. Evoked potential studies c. Electromyography (EMG) d. Electroencephalography (EEG)

ANS: D Seizure disorders are usually studied using EEG testing.

When assessing a patient with bacterial meningitis, the nurse obtains the following data. Which finding should be reported immediately to the health care provider? a. The patient has a positive Kernig's sign. b. The patient complains of having a stiff neck. c. The patient's temperature is 101° F (38.3° C). d. The patient's blood pressure is 86/42 mm Hg.

ANS: D Shock is a serious complication of meningitis, and the patient's low blood pressure indicates the need for interventions such as fluids or vasopressors. Nuchal rigidity and a positive Kernig's sign are expected with bacterial meningitis. The nurse should intervene to lower the temperature, but this is not as life threatening as the hypotension.

When performing a focused assessment on a patient with a lesion of the left posterior temporal lobe, the nurse will assess for a. sensation on the left side of the body. b. voluntary movement on the right side. c. reasoning and problem-solving abilities. d. understanding of written and oral language.

ANS: D The posterior temporal lobe integrates the visual and auditory input for language comprehension. Reasoning and problem solving are functions of the anterior frontal lobe. Sensation on the left side of the body is located in the right postcentral gyrus. Voluntary movement on the right side is controlled in the left precentral gyrus.

Which assessment information will the nurse collect to determine whether a patient is developing postconcussion syndrome? a. Muscle resistance b. Short-term memory c. Glasgow coma scale d. Pupil reaction to light

B Short term memory

NCLEX review questions: A 59-year-old female patient, who has frontotemporal lobar degeneration, has difficulty with verbal expression. One day she walks out of the house and goes to the gas station to get a soda but does not understand that she needs to pay for it. What is the best thing the nurse can suggest to this patient's husband to keep the patient safe during the day while the husband is at work? A) Assisted living B) Adult day care C) Advance directives D) Monitor for behavioral changes

B) Adult day care Rationale: To keep this patient safe during the day while the husband is at work, an adult day care facility would be the best choice. This patient would not need assisted living. Advance directives are important but are not related to her safety. Monitoring for behavioral changes will not keep her safe during the day.

NCLEX review questions: When providing community health care teaching regarding the early warning signs of Alzheimer's disease, which signs should the nurse advise family members to report (select all that apply)? A) Misplacing car keys B) Losing sense of time C) Difficulty performing familiar tasks D) Problems with performing basic calculations E) Becoming lost in a usually familiar environment

B) Losing sense of time C) Difficulty performing familiar tasks D) Problems with performing basic calculations E) Becoming lost in a usually familiar environment Rationale: Difficulty performing familiar tasks, problems with performing basic calculations, losing sense of time, and becoming lost in a usually familiar environment are all part of the early warning signs of Alzheimer's disease. Misplacing car keys is a normal frustrating event for many people.

To promote communication during rehabilitation of the patient with aphasia, an appropriate nursing intervention is to a. use gestures, pictures, and music to stimulate patient responses b. talk about activities of daily living (ADLs) that are familiar to the patient c. structure statements so that patient does not have to respond verbally d. use flashcards with simple words and pictures to promote language recall

B: Talk about ADLs that are familiar to the patient- during rehabilitation, the patient with aphasia needs frequent, meaningful verbal stimulation that has relevance for him. Conversation by the nurse and family should address ADLs that are familiar to the patient. Gestures, pictures, and simple statements are more appropriate in the acute phase, when patients may be overwhelmed with verbal stimuli. Flashcards are often perceived by the patient as childish and meaningless.

In planning long-term care for a patient after a craniotomy, what must the nurse include when teaching the patient, family, and caregiver? A. Seizure disorders may occur in weeks or months. B. The family will be unable to cope with role reversals. C. There are often residual changes in personality and cognition. D. Referrals will be made to eliminate residual deficits from the damage.

C In long-term care planning, the nurse must include the family and caregiver when teaching about potential residual changes in personality, emotions, and cognition as these changes are most difficult for the patient and family to accept. Seizures may or may not develop. The family and patient may or may not be able to cope with role reversals. Although residual deficits will not be eliminated with referrals, they may be improved.

The nurse prepares to administer temozolomide (Temodar) to a 59-year-old white male patient with a glioblastoma multiforme (GBM) brain tumor. What should the nurse assess before giving the medication? A. Serum potassium and serum sodium levels B. Urine osmolality and urine specific gravity C. Absolute neutrophil count and platelet count D. Cerebrosprinal fluid (CSF) pressure and cell count

C Temozolomide causes myelosuppression. The nurse should assess the absolute neutrophil count and the platelet count. The absolute neutrophil count should be >1500/μL and platelet count >100,000/μL.

NCLEX review questions: Benzodiazepines are indicated in the treatment of cases of delirium that have which cause? A) Polypharmacy B) Cerebral hypoxia C) Alcohol withdrawal D) Electrolyte imbalances

C) Alcohol withdrawal Rationale: Benzodiazepines can be used to treat delirium associated with sedative and alcohol withdrawal. However, these drugs may worsen delirium caused by other factors and must be used cautiously. Polypharmacy, cerebral hypoxia, and electrolyte imbalances are not treated with benzodiazepines.

NCLEX review questions: Which nursing intervention is most appropriate when caring for patients with dementia? Avoid direct eye contact. Lovingly call the patient "honey" or "sweetie." Give simple directions, focusing on one thing at a time. Treat the patient according to his or her age-related behavior.

C) Give simple directions, focusing on one thing at a time. Rationale: When dealing with patients with dementia, tasks should be simplified, giving directions using gestures or pictures and focusing on one thing at a time. It is best to treat these patients as adults, with respect and dignity, even when their behavior is childlike. The nurse should use gentle touch and direct eye contact. Calling the patient "honey" or "sweetie" can be condescending and does not demonstrate respect.

Pre-test practice questions: Unlicensed assistive personnel (UAP) working for a home care agency report a change in the alertness and language of an 82-year-old female patient. The home care nurse plans a visit to evaluate the patient's cognitive function. Which assessment would be most appropriate? A) Glasgow Coma Scale (GCS) B) Confusion Assessment Method (CAM) C) Mini-Mental State Examination (MMSE) D) National Institutes of Health Stroke Scale (NIHSS)

C) Mini-Mental State Examination (MMSE) Rationale: The MMSE is a commonly used tool to assess cognitive function. Cognitive testing is focused on evaluating memory, ability to calculate, language, visual-spatial skills, and degree of alertness. The CAM is used to assess for delirium. The GCS is used to assess the degree of impaired consciousness. The NIHSS is a neurologic examination stroke scale used to evaluate the effect of acute cerebral infarction on the levels of consciousness, language, neglect, visual field loss, extraocular movement, motor strength, ataxia, dysarthria, and sensory loss.

An unconscious patient with increased ICP is on ventilatory support. The nurse notifies the health care provider when arterial blood gas measurement results reveal a a. pH of 7.43 b. SaO2 of 94% c. PaO2 of 50 mm Hg d. PaCO2 of 30 mm Hg

C. A PaO2 of 50 mm Hg reflects a hypoxemia that may lead to further decreased cerebral perfusion and hypoxia and must be corrected. The pH of SaO2 are within normal range, and a PaCO2 of 30 mm Hg reflects acceptable value for the patient with increased ICP

Increased ICP in the left cerebral cortex, caused by intracranial bleeding causes displacement of brain tissue to the right hemisphere beneath the falx cerebri. The nurse knows that this is referred to as a. uncal herniation b. tentorial herniation c. cingulate herniation d. temporal lobe herniation

C. Cingulate herniation- the dural structures that separate the two hemispheres and the cerebral hemispheres from the cerebellum influence the patterns of cerebral herniation. A cingulated herniation occurs where there is lateral displacement of brain tissue beneath the falx cerebri.

While the nurse performs ROM on an unconscious patient with increased ICP, the patient experiences severe decerebrate posturing reflexes. The nurse should a. use restraints to protect the patient from injury b. administer CNS depressants to lightly sedate the patient c. perform the exercises less frequently because posturing can increase ICP d. continue the exercises because they are necessary to maintain musculoskeletal function

C. If reflex posturing occurs during ROM or positioning of the patient, these activities should be done less frequently until the patient's condition stabilizes, because posturing can case increases in ICP. Neither restraints nor CNS depressants would be indicated.

CN III originating in the midbrain is assessed by the nurse for an early indication of pressure on the brainstem by a. assessing for nystagmus b. testing the corneal reflex c. testing pupillary reaction to light d. testing for oculocephalic (doll's eye) reflex

C. One of the functions of CN III, the oculomotor nerve, is pupillary constriction, and testing for pupillary constriction is important to identify patients at risk for brainstem herniation caused by increased ICP. The corneal reflex is used to assess the functions of CN V and VII, and the oculocephalic reflex tests all cranial nerves involved with eye movement. Nystagmus is commonly associatted with specific lesions or chemical toxicities and is not a definitive sign of ICP.

The nurse is alerted to a possible acute subdural hematoma in the patient who a. Has linear skull fracture crossing a major artery b. Has focal symptoms of brain damage with no recollection of a head injury c. Develops decreased level of consciousness and a headache within 48 hrs of a head injury d. Has an immediate loss of consciousness with brief lucid interval followed by dereasing level of consciousness

Correct answer: c Rationale: An acute subdural hematoma manifests within 24 to 48 hours of the injury. The signs and symptoms are similar to those associated with brain tissue compression in elevated ICP and include decreasing level of consciousness and headache

A patient has a systemic BP of 108/51 mm Hg and an intracranial pressure (ICP) of 14 mm Hg. Which action should the nurse take first? a. Elevate the head of the patient's bed to 60 degrees. b. Document the BP and ICP in the patient's record. c. Report the BP and ICP to the health care provider. d. Continue to monitor the patient's vital signs and ICP.

ANS: C The patient's cerebral perfusion pressure is 56 mm Hg, below the normal of 60 to 100 mm Hg and approaching the level of ischemia and neuronal death. Immediate changes in the patient's therapy such as fluid infusion or vasopressor administration are needed to improve the cerebral perfusion pressure. Adjustments in the head elevation should only be done after consulting with the health care provider. Continued monitoring and documentation also will be done, but they are not the first actions that the nurse should take.

The nurse performing a focused assessment of left posterior temporal lobe functions will assess the patient for a. sensation on the left side of the body. b. reasoning and problem-solving ability. c. ability to understand written and oral language. d. voluntary movements on the right side of the body.

ANS: C The posterior temporal lobe integrates the visual and auditory input for language comprehension. Reasoning and problem solving are functions of the anterior frontal lobe. Sensation on the left side of the body is located in the right postcentral gyrus. Voluntary movement on the right side is controlled in the left precentral gyrus.

Which hospitalized patient will the nurse assign to the room closest to the nurses' station? a. Patient with Alzheimer's disease who has long-term memory deficit b. Patient with vascular dementia who takes medications for depression c. Patient with new-onset confusion, restlessness, and irritability after surgery d. Patient with dementia who has an abnormal Mini-Mental State Examination

ANS: C This patient's history and clinical manifestations are consistent with delirium. The patient is at risk for safety problems and should be placed near the nurses' station for ongoing observation. The other patients have chronic symptoms that are consistent with their diagnoses but are not at immediate risk for safety issues.

A 32-year-old female patient is diagnosed with diabetes insipidus after transsphenoidal resection of a pituitary adenoma. What should the nurse consider as a sign of improvement? A. Serum sodium of 120 mEq/L B. Urine specific gravity of 1.001 C. Fasting blood glucose of 80 mg/dL D. Serum osmolality of 290 mOsm/kg

D Laboratory findings in diabetes insipidus include an elevation in serum osmolality and serum sodium and a decrease in urine specific gravity. Normal serum osmolality is 275 to 295 mOsm/kg, normal serum sodium is 135 to 145 mEq/L, and normal specific gravity is 1.003 to 1.030. Elevated blood glucose levels occur with diabetes mellitus.

The priority intervention in the emergency department for the patient with a stroke is a. intravenous fluid replacement b. administration of osmotic diuretics to reduce cerebral edema c. initiation of hypothermia to decrease the oxygen needs of the brain d. maintenance of respiratory function with a patent airway and oxygen administration

D: Maintenance of respiratory function with a patent airway and oxygen administration- the first priority in acute management of the patient with a stroke is preservation of life. Because the patient with a stroke may be unconscious or have a reduced gag reflex, it is most important to maintain a patent airway for the patient and provide oxygen if respiratory effort is impaired. IV fluid replacement, treatment with osmotic diuretics, and perhaps hypothermia may be used for further treatment.

intracerebral hemorrhage

a type of hemorrhagic stroke in which bleeding within the brain caused by a rupture of a blood vessel occurs; often caused by hypertension and is associated with increased intracranial pressure

Common psychosocial reactions of the stroke patient to the stroke include (select all that apply) a. depression. b. disassociation. c. intellectualization. d. sleep disturbances. e. denial of severity of stroke.

a, d, e Rationale: The patient with a stroke may experience many losses, including sensory, intellectual, communicative, functional, role behavior, emotional, social, and vocational losses. Some patients experience long-term depression, manifesting symptoms such as anxiety, weight loss, fatigue, poor appetite, and sleep disturbances. The time and energy required to perform previously simple tasks can result in anger and frustration. Frustration and depression are common in the first year after a stroke. A stroke is usually a sudden, extremely stressful event for the patient, caregiver, family, and significant others. The family is often affected emotionally, socially, and financially as their roles and responsibilities change. Reactions vary considerably but may involve fear, apprehension, denial of the severity of stroke, depression, anger, and sorrow.

What manifestations of cognitive impairment are primarily characteristic of delirium (select all that apply)? a. Reduced awareness b. Impaired judgments c. Words difficult to find d. Sleep/wake cycle reversed e. Distorted thinking and perception f. Insidious onset with prolonged duration

a, d, e. Manifestations of delirium include cognitive impairment with reduced awareness, reversed sleep/wake cycle, and distorted thinking and perception. The other options are characteristic of dementia.

6. When administering a mental status examination to a patient, the nurse suspects depression when the patient responds with a. "I don't know." b. "Is that the right answer?" c. "Wait, let me think about that." d. "Who are those people over there?

a. "I don't know." Answers such as "I don't know" are more typical of depression. The response "Who are those people over there?" is more typical of the distraction seen in a patient with delirium. The remaining two answers are more typical of a patient with dementia.

11. A patient with mild dementia has a new prescription for donepezil (Aricept). Which nursing action will be most effective in ensuring compliance with the medication? a. Having the patient's spouse administer the medication b. Setting the medications up weekly in a medication box c. Calling the patient daily with a reminder to take the medication d. Posting reminders to take the medications in the patient's house

a. Having the patient's spouse administer the medication Because the patient with mild dementia will have difficulty with learning new skills and forgetfulness, the most appropriate nursing action is to have someone else administer the drug. The other nursing actions will not be as effective in ensuring that the patient takes the medications.

The newly admitted patient has moderate AD. What does the nurse know this patient will need help with? a. Eating b. Walking c. Dressing d. Self-care activities

c. In the moderate stage of AD, the patient may need help with getting dressed. In the severe stage, patients will be unable to dress or feed themselves and are usually incontinent.

14. During the morning change-of-shift report at the long-term care facility, the nurse learns that the patient with dementia has had sundowning. Which nursing action should the nurse take while caring for the patient? a. Provide hourly orientation to time of day. b. Move the patient to a quieter room at night. c. Keep blinds open during the daytime hours. d. Have the patient take a brief mid-morning nap.

c. Keep blinds open during the daytime hours. The most likely cause of sundowning is a disruption in circadian rhythms and keeping the patient active and in daylight will help to reestablish a more normal circadian pattern. Moving the patient to a different room might increase confusion. Taking a nap will interfere with nighttime sleep. Hourly orientation will not be helpful in a patient with memory difficulties.

10. When teaching the children of a patient who is being evaluated for Alzheimer's disease (AD) about the disorder, the nurse explains that a. the most important risk factor for AD is a family history of the disorder. b. new drugs have been shown to reverse AD dramatically in some patients. c. a diagnosis of AD can be made only when other causes of dementia have been ruled out. d. the presence of brain atrophy detected by MRI confirms the diagnosis of AD in patients with dementia.

c. a diagnosis of AD can be made only when other causes of dementia have been ruled out. The diagnosis of AD is one of exclusion. Age is the most important risk factor for development of AD. Drugs can slow the deterioration but do not dramatically reverse the effects of AD. Brain atrophy is a common finding in AD, but it can occur in other diseases as well and does not confirm an AD diagnosis.

15. A long-term care patient with moderate dementia develops increased restlessness and agitation. The nurse's initial action should be to a. reorient the patient to time, place, and person. b. administer the PRN dose of lorazepam (Ativan). c. assess for factors that might be causing discomfort. d. have a nursing assistant stay with the patient to ensure safety.

c. assess for factors that might be causing discomfort. Increased motor activity in a patient with dementia is frequently the patient's only way of responding to factors like pain, so the nurse's initial action should be to assess the patient for any precipitating factors. Administration of sedative drugs may be indicated, but this should not be done until assessment for precipitating factors has been completed and any of these factors have been addressed. Reorientation is unlikely to be helpful for the patient with moderate dementia. Assigning a nursing assistant to stay with the patient also may be necessary, but any physical changes that may be causing the agitation should be addressed first.

Bridge to NCLEX questions: Vascular dementia is associated with a. transient ischemic attacks. b. bacterial or viral infection of neuronal tissue. c. cognitive changes secondary to cerebral ischemia. d. abrupt changes in cognitive function that are irreversible.

c. cognitive changes secondary to cerebral ischemia. Rationale: Vascular dementia is the loss of cognitive function that results from ischemic, ischemic-hypoxic, or hemorrhagic brain lesions caused by cardiovascular disease. In this type of dementia, narrowing and blocking of arteries that supply the brain causes a decrease in blood supply.

aneurysms

congenital or acquired weakness of the arterial wall resulting in dilation and ballooning of the vessel

Information provided by the patient that would help differentiate a hemorrhagic stroke from a thrombotic stroke includes a. sensory disturbance. b. a history of hypertension. c. presence of motor weakness. d. sudden onset of severe headache.

d Rationale: A hemorrhagic stroke usually causes a sudden onset of symptoms, which include neurologic deficits, headache, nausea, vomiting, decreased level of consciousness, and hypertension. Ischemic stroke symptoms may progress in the first 72 hours as infarction and cerebral edema increase.

Assessment of pupil response to light and ptosis are used to check function of the

oculomotor nerve.

ischemic stroke

stroke that results from inadequate blood flow to the brain caused by partial or complete occlusion of an artery

cerebrovascular accident (CVA)

term used to describe a stroke

brain attack

term used to describe a stroke; communicates the urgency of recognizing the clinical manifestations of a stroke and treating a medical emergency

Evoked potential is used for diagnosing problems with .

the visual or auditory systems

Cerebral angiography is used to diagnose

vascular problems.

dysarthria

a disturbance in the muscular control of speech, resulting from interference in the control and execution over the muscles of speech, usually caused by damage to a central or peripheral motor nerve

subarachnoid hemorrhage

a stroke resulting from intracranial bleeding into the cerebrospinal fluid-filled space between the arachnoid and pia mater membranes on the surface of the brain

thrombotic stroke

a stroke resulting from thrombosis or narrowing of the blood vessel

embolic stroke

a stroke that occurs when an embolus lodges in and occludes a cerebral artery, resulting in infarction and edema of the area supplied by the involved vessel

14. The son of a patient with early-onset AD asks if he will get AD. What should the nurse tell this man about the genetics of AD? a. The risk of early-onset AD for the children of parents with it is about 50%. b. Women get AD more often than men do, so his chances of getting AD are slim. c. The blood test for the ApoE gene to identify this type of AD can predict who will develop it. d. This type of AD is not as complex as regular AD, so he does not need to worry about getting AD.

14. a. The risk of early-onset AD for the children of parents with it is 50%. Women do get AD more often than men but that is more likely related to women living longer than men than to the type of AD. ApoE gene testing is used for research with late-onset AD but does not predict who will develop the disease. Late-onset AD is more genetically complex than early-onset AD and is more common in those over age 60 but because his parent has early-onset AD he is at a 50% risk of getting it.

15. A patient with moderate AD has a nursing diagnosis of impaired memory related to effects of dementia. What is an appropriate nursing intervention for this patient? a. Post clocks and calendars in the patient's environment. b. Establish and consistently follow a daily schedule with the patient. c. Monitor the patient's activities to maintain a safe patient environment. d. Stimulate thought processes by asking the patient questions about recent activities

15. b. Adhering to a regular, consistent daily schedule helps the patient to avoid confusion and anxiety and is important both during hospitalization and at home. Clocks and calendars may be useful in early AD but they have little meaning to a patient as the disease progresses. Questioning the patient about activities and events they cannot remember is threatening and may cause severe anxiety. Maintaining a safe environment for the patient is important but does not change the disturbed thought processes.

17. The wife of a man with moderate AD has a nursing diagnosis of social isolation related to diminishing social relationships and behavioral problems of the patient with AD. What is a nursing intervention that would be appropriate to provide respite care and allow the wife to have satisfactory contact with significant others? a. Help the wife to arrange for adult day care for the patient. b. Encourage permanent placement of the patient in the Alzheimer's unit of a long-term care facility. c. Refer the wife to a home health agency to arrange daily home nursing visits to assist with the patient's care. d. Arrange for hospitalization of the patient for 3 or 4 days so that the wife can visit out-of-town friends and relatives.

17. a. Adult day care is an option to provide respite for caregivers and a protective environment for the patient during the early and middle stages of AD. There are also in-home respite care providers. The respite from the demands of care allows the caregiver to maintain social contacts, perform normal tasks of living, and be more responsive to the patient's needs. Visits by home health nurses involve the caregiver and cannot provide adequate respite. Institutional placement is not always an acceptable option at earlier stages of AD, nor is hospitalization available for respite care.

18. The health care provider is trying to differentiate the diagnosis of the patient between dementia and dementia with Lewy bodies (DLB). What observations by the nurse support a diagnosis of DLB (select all that apply)? a. Tremors b. Fluctuating cognitive ability c. Disturbed behavior, sleep, and personality d. Symptoms of pneumonia, including congested lung sounds e. Bradykinesia, rigidity, and postural instability without tremor

18. b, e. Dementia with Lewy bodies (DLB) is diagnosed with dementia plus two of the following symptoms: (1) extrapyramidal signs such as bradykinesia, rigidity, and postural instability but not always a tremor, (2) fluctuating cognitive ability, and (3) hallucinations. The extrapyramidal signs plus tremors would more likely indicate Parkinson's disease. Disturbed behavior, sleep, personality, and eventually memory are characteristics of frontotemporal lobe degeneration (FTLD).

19. Delegation Decision: The RN in charge at a long-term care facility could delegate which activities to unlicensed assistive personnel (UAP) (select all that apply)? a. Assist the patient with eating. b. Provide personal hygiene and skin care. c. Check the environment for safety hazards. d. Assist the patient to the bathroom at regular intervals. e. Monitor for skin breakdown and swallowing difficulties.

19. a, b, d. All caregivers are responsible for the patient's safety. Basic care activities, such as those associated with personal hygiene and activities of daily living (ADLs) can be delegated to unlicensed assistive personnel (UAP). The RN will perform ongoing assessments and develop and revise the plan of care as needed. The RN will assess the patient's safety risk factors, provide education, and make referrals. The licensed practical nurse (LPN) could check the patient's environment for potential safety hazards.

2. Which statement accurately describes dementia? a. Overproduction of β-amyloid protein causes all dementias. b. Dementia resulting from neurodegenerative causes can be prevented. c. Dementia caused by hepatic or renal encephalopathy cannot be reversed. d. Vascular dementia can be diagnosed by brain lesions identified with neuroimaging.

2. d. The diagnosis of vascular dementia can be aided by neuroimaging studies showing vascular brain lesions along with exclusion of other causes of dementia. Overproduction of β-amyloid protein contributes to Alzheimer's disease (AD). Vascular dementia can be prevented or slowed by treating underlying diseases (e.g., diabetes mellitus, cardiovascular disease). Dementia caused by hepatic or renal encephalopathy potentially can be reversed.

20. A 72-year-old woman is hospitalized in the intensive care unit (ICU) with pneumonia resulting from chronic obstructive pulmonary disease (COPD). She has a fever, productive cough, and adventitious breath sounds throughout her lungs. In the past 24 hours her fluid intake was 1000 mL and her urine output was 700 mL. She was diagnosed with early-stage AD 6 months ago but has been able to maintain her activities of daily living (ADLs) with supervision. Identify at least six risk factors for the development of delirium in this patient. (Fill in the blanks.) a. b. c. d. e. f.

20. a. Age; b. infection; c. hypoxemia (lung disease); d. intensive care unit (ICU) hospitalization (change in environment, sensory overload); e. preexisting dementia; f. dehydration. Also: hyperthermia and potentially medications to treat chronic obstructive pulmonary disease (COPD) and pneumonia.

21. A 68-year-old man is admitted to the emergency department with multiple blunt trauma following a one-vehicle car accident. He is restless; disoriented to person, place, and time; and agitated. He resists attempts at examination and calls out the name "Janice." Why should the nurse suspect delirium rather than dementia in this patient? a. The fact that he wouldn't have been allowed to drive if he had dementia b. His hyperactive behavior, which differentiates his condition from the hypoactive behavior of dementia c. The report of emergency personnel that he was noncommunicative when they arrived at the accident scene d. The report of his family that although he has heart disease and is "very hard of hearing," this behavior is unlike him

21. d. Delirium is an acute problem that usually has a rapid onset in response to a precipitating event, especially when the patient has underlying health problems, such as heart disease and sensory limitations. In the absence of prior cognitive impairment, a sudden onset of confusion, disorientation, and agitation is usually delirium. Delirium may manifest with both hypoactive and hyperactive symptoms.

What should be included in the management of a patient with delirium? a. The use of restraints to protect the patient from injury b. The use of short-acting benzodiazepines to sedate the patient c. Identification and treatment of underlying causes when possible d. Administration of high doses of an antipsychotic drug such as haloperidol (Haldol)

22. c. Care of the patient with delirium is focused on identifying and eliminating precipitating factors if possible. Treatment of underlying medical conditions, changing environmental conditions, and discontinuing medications that induce delirium are important. Drug therapy is reserved for those patients with severe agitation because the drugs themselves may worsen delirium.

23. When caring for a patient in the severe stage of AD, what diversion or distraction activities would be appropriate? a. Watching TV b. Playing games c. Books to read d. Mobiles or dangling ribbons

23. d. In the severe stage of AD, the patient is at a developmental level of 15 months or less; therefore appropriate distractions would be infant toys. Watching TV and playing games are more appropriate in the mild stage. Books to read would need to be at developmentally appropriate levels to be used as a diversion.

3. A patient with Alzheimer's disease (AD) dementia has manifestations of depression. The nurse knows that treatment of the patient with antidepressants will most likely do what? a. Improve cognitive function b. Not alter the course of either condition c. Cause interactions with the drugs used to treat the dementia d. Be contraindicated because of the central nervous system (CNS)-depressant effect of antidepressants

3. a. Depression is often associated with AD, especially early in the disease when the patient has awareness of the diagnosis and the progression of the disease. When dementia and depression occur together, intellectual deterioration may be more extreme. Depression is treatable and use of antidepressants often improves cognitive function.

4. For what purpose would the nurse use the Mini-Mental State Examination to evaluate a patient with cognitive impairment? a. It is a good tool to determine the etiology of dementia. b. It is a good tool to evaluate mood and thought processes. c. It can help to document the degree of cognitive impairment in delirium and dementia. d. It is useful for initial evaluation of mental status but additional tools are needed to evaluate changes in cognition over time.

4. c. The Mini-Mental State Examination is a tool to document the degree of cognitive impairment and it can be used to determine a baseline from which changes over time can be evaluated. It does not evaluate mood or thought processes but can detect dementia and delirium and differentiate these from psychiatric mental illness. It cannot help to determine etiology.

5. During assessment of a patient with dementia, the nurse determines that the condition is potentially reversible when finding out what about the patient? a. Has long-standing abuse of alcohol b. Has a history of Parkinson's disease c. Recently developed symptoms of hypothyroidism d. Was infected with human immunodeficiency virus (HIV) 10 years ago

5. c. Hypothyroidism can cause dementia but it is a treatable condition if it has not been long standing. The other conditions are causes of irreversible dementia.

6. The husband of a patient is complaining that his wife's memory has been decreasing lately. When asked for examples of her memory loss, the husband says that she is forgetting the neighbors' names and forgot their granddaughter's birthday. What kind of loss does the nurse recognize this to be? a. Delirium b. Memory loss in AD c. Normal forgetfulness d. Memory loss in mild cognitive impairment

6. d. In mild cognitive impairment people frequently forget people's names and begin to forget important events. Delirium changes usually occur abruptly. In Alzheimer's disease the patient may not remember knowing a person and loses the sense of time and which day it is. Normal forgetfulness includes momentarily forgetting names and occasionally forgetting to run an errand.

7. The wife of a patient who is manifesting deterioration in memory asks the nurse whether her husband has AD. The nurse explains that a diagnosis of AD is usually made when what happens? a. A urine test indicates elevated levels of isoprostanes b. All other possible causes of dementia have been eliminated c. Blood analysis reveals increased amounts of β-amyloid protein d. A computed tomography (CT) scan of the brain indicates brain atrophy

7. b. The only definitive diagnosis of AD can be made on examination of brain tissue during an autopsy but a clinical diagnosis is made when all other possible causes of dementia have been eliminated. Patients with AD may have β-amyloid proteins in the blood, brain atrophy, or isoprostanes in the urine but these findings are not exclusive to those with AD.

9. What is one focus of collaborative care of patients with AD? a. Replacement of deficient acetylcholine in the brain b. Drug therapy for cognitive problems and undesirable behaviors c. The use of memory-enhancing techniques to delay disease progression d. Prevention of other chronic diseases that hasten the progression of AD

9. b. Because there is no cure for AD, collaborative management is aimed at controlling the decline in cognition, controlling the undesirable manifestations that the patient may exhibit, and providing support for the family caregiver. Anticholinesterase agents help to increase acetylcholine (ACh) in the brain but a variety of other drugs are also used to control behavior. Memoryenhancing techniques have little or no effect in patients with AD, especially as the disease progresses. Patients with AD have limited ability to communicate health symptoms and problems, leading to a lack of professional attention for acute and other chronic illnesses.

Pre-test practice questions: Although he has been told that ginkgo biloba will probably have no effect, a 58-year-old man with early stage Alzheimer's disease insists on taking the herb because he believes it will slow the disease progression. Which statement, if made by the patient to the nurse, indicates understanding about the side effects of ginkgo? A) "Ginkgo may increase the risk of bruising." B) "Ginkgo may cause leg pain while walking." C) "It is not safe to suddenly stop taking ginkgo." D) "Ringing in the ears is a side effect of ginkgo."

A) "Ginkgo may increase the risk of bruising." Rationale: Ginkgo biloba may increase the risk for bruising and bleeding. There are no indications that sudden withdrawal of ginkgo biloba is unsafe. Ginkgo biloba is possibly effective for treating intermittent claudication (leg pain while walking). There is insufficient evidence to indicate that ginkgo biloba is effective in treatment of tinnitus (ringing in the ears).

NCLEX review questions: Which statement by the wife of a patient with Alzheimer's disease (AD) demonstrates an accurate understanding of her husband's medication regimen? A) "I'm really hoping his medications will slow down his mental losses." B) "We're both holding out hope that this medication will cure his disease." C) "I know that this won't cure him, but we learned that it might prevent a bodily decline while he declines mentally." D) "I learned that if we are vigilant about his medication schedule, he may not experience the physical effects of his disease."

A) "I'm really hoping his medications will slow down his mental losses." Rationale: There is presently no cure for Alzheimer's disease, and drug therapy aims at improving or controlling decline in cognition. Medications do not directly address the physical manifestations of AD.

NCLEX review questions: For which patient should the nurse prioritize an assessment for depression? A) A patient in the early stages of Alzheimer's disease B) A patient who is in the final stages of Alzheimer's disease C) A patient experiencing delirium secondary to dehydration D) A patient who has become delirious following an atypical drug response

A) A patient in the early stages of Alzheimer's disease Rationale: Patients in the early stages of Alzheimer's disease are particularly susceptible to depression, since the patient is acutely aware of his or her cognitive changes and the expected disease trajectory. Delirium is typically a shorter-term health problem that does not typically pose a heightened risk of depression.

NCLEX review questions: The nurse who has administered a dose of risperidone (Risperdal) to a patient with delirium should assess for what intended effect of the medication? A) Lying quietly in bed B) Alleviation of depression C) Reduction in blood pressure D) Disappearance of confusion

A) Lying quietly in bed Rationale: Risperidone is an antipsychotic drug that reduces agitation and produces a restful state in patients with delirium. However, it should be used with caution. Antidepressant medications treat depression, and antihypertensive medications treat hypertension. However, there are no medications that will cause confusion to disappear in a patient with delirium.

Pre-test practice questions: A 78-year-old woman is in the intensive care unit after emergency abdominal surgery. The nurse notes that the patient is disoriented and confused, has incoherent speech, and is restless and agitated. Which action by the nurse is most appropriate? A) Reorient the patient. B) Notify the physician. C) Document the findings. D) Administer lorazepam (Ativan).

A) Reorient the patient. Rationale: The patient is exhibiting clinical manifestations of delirium. Care of the patient with delirium is focused on eliminating precipitating factors and protecting the patient from harm. Give priority to creating a calm and safe environment. The nurse should stay at the bedside and provide reassurance and reorienting information as to place, time, and procedures. The nurse should reduce environmental stimuli, including noise and light levels. Avoid the use of chemical and physical restraints if possible.

NCLEX review questions: The patient is having some increased memory and language problems. What diagnostic tests will be done before this patient is diagnosed with Alzheimer's disease (select all that apply)? A) Urinalysis B) MRI of the head C) Liver function tests D) Neuropsychologic testing E) Blood urea nitrogen and serum creatinine

A) Urinalysis B) MRI of the head C) Liver function tests D) Neuropsychologic testing E) Blood urea nitrogen and serum creatinine Rationale: Because there is no definitive diagnostic test for Alzheimer's disease, and many conditions can cause manifestations of dementia, testing must be done to eliminate any other causes of cognitive impairment. These include urinalysis to eliminate a urinary tract infection, an MRI to eliminate brain tumors, liver function tests to eliminate encephalopathy, BUN and serum creatinine to rule out renal dysfunction, and neuropsychologic testing to assess cognitive function.

The nurse observes a student nurse assigned to initiate oral feedings for a 68-year-old woman with an ischemic stroke. The nurse should intervene if she observes the student nurse: A. giving the patient 8 ounces of ice water to swallow. B. telling the patient to perform a chin tuck before swallowing. C. assisting the patient to sit in a chair before feeding the patient. D. assessing cranial nerves IX and X before the patient attempts to eat.

A. giving the patient 8 ounces of ice water to swallow. The majority of patients after a stroke have dysphagia. The gag reflex and swallowing ability (cranial nerves IX and X) should be assessed before the first oral feeding. To assess swallowing ability, the nurse should elevate the head of the bed to an upright position (unless contraindicated) and give the patient a very small amount (not 8 ounces) of crushed ice or ice water to swallow. The patient should remain in a high Fowler's position, preferably in a chair with the head flexed forward, for the feeding and for 30 minutes following.

A patient is admitted to the hospital with a left hemiplegia. To determine the size and location and to ascertain whether a stroke is ischemic or hemorrhagic, the nurse anticipates that the health care provider will request a a. CT scan b. lumbar puncture c. cerebral arteriogram d. positron emission tomography (PET)

A: CT scan- A CT scan is the most commonly used diagnostic test to determine the size and location of the lesion and to differentiate a thrombotic stroke from a hemorrhagic stroke. Positron emission tomography (PET) will show the metabolic activity of the brain and provide a depiction of the extent of tissue damage after a stroke. Lumbar punctures are not performed routinely because of the chance of increased intracranial pressure causing herniation. Cerebral arteriograms are invasive and may dislodge an embolism or cause further hemorrhage; they are performed only when no other test can provide the needed information.

During the acute phase of a stroke, the nurse assesses the patient's vital signs and neurologic status every 4 hours. A cardiovascular sign that the nurse would see as the body attempts to increase cerebral blood flow is a. hypertension b. fluid overload c. cardiac dysrhythmias d. S3 and S4 heart sounds

A: Hypertension- The body responds to the vasopasm and a decreased circulation to the brain that occurs with a stroke by increasing the BP, frequently resulting in hypertension. The other options are important cardiovascular factors to assess, but they do not result from impaired cerebral blood flow.

A newly admitted patient who has suffered a right sided brain stroke has a nursing diagnosis of disturbed visual sensory perception related to homonymous hemianopsia. Early in the care of the patient, the nurse should a. place objects on the right side within the patient's field of vision b. approach the patient from the left side to encourage the patient to turn the head c. place objects on the patient's left side to assess the patient's ability to compensate d. patch the affected eye to encourage the patient to turn the head to scan the environment

A: Place objects on the right side within the patient's field of vision- the presence of homonymous hemianopia in a patient with right-hemisphere brain damage causes a loss of vision in the left field. Early in the care of the patient, objects should be placed on the right side of the patient in the field of vision, and the nurse should approach the patient from the right side. Later in treatment, patients should be taught to turn the head and scan the environment and should be approached from the affected side to encourage head turning. Eye patches are used if patients have diplopia (double vision).

Four days following a stroke, a patient is to start oral fluids and feedings. Before feeding the patient, the nurse should first a. check the patient's gag reflex b. order a soft diet for the patient c. raise the head of the bed to sitting position d. evaluate the patient's ability to swallow small sips of ice water

A: check the patient's gag reflex- the first step in providing oral feedings for a patient with a stroke is ensuring that the patient has an intact gag reflex because oral feedings will not be provided if gag reflex is impaired. The nurse should then evaluate the patient's ability to swallow ice chips or ice water after placing the patient in an upright position

The day shift nurse at the long-term care facility learns that a patient with dementia experienced sundowning late in the afternoon on the previous two days. Which action should the nurse take? a. Keep blinds open during the daytime hours. b. Provide hourly orientation to time and place. c. Have the patient take a brief mid-morning nap. d. Move the patient to a quieter room late in the afternoon.

ANS: A A likely cause of sundowning is a disruption in circadian rhythms and keeping the patient active and in daylight will help reestablish a more normal circadian pattern. Moving the patient to a different room might increase confusion. Taking a nap will interfere with nighttime sleep. Hourly orientation will not be helpful in a patient with dementia

A patient is brought to the emergency department (ED) by ambulance after being found unconscious on the bathroom floor by the spouse. Which action will the nurse take first? a. Obtain oxygen saturation. b. Check pupil reaction to light. c. Palpate the head for hematoma. d. Assess Glasgow Coma Scale (GCS).

ANS: A Airway patency and breathing are the most vital functions and should be assessed first. The neurologic assessments should be accomplished next and the health and medication history last.

A patient has a tumor in the cerebellum. The nurse will plan interventions to a. prevent falls. c. avoid aspiration. b. stabilize mood. d. improve memory.

ANS: A Because functions of the cerebellum include coordination and balance, the patient with dysfunction is at risk for falls. The cerebellum does not affect memory, mood, or swallowing ability.

When developing a plan of care for a patient with dysfunction of the cerebellum, the nurse will include interventions to a. prevent falls. b. stabilize mood. c. enhance swallowing ability. d. improve short-term memory.

ANS: A Because functions of the cerebellum include coordination and balance, the patient with dysfunction is at risk for falls. The cerebellum does not affect memory, mood, or swallowing ability.

When a patient's intracranial pressure (ICP) is being monitored with an intraventricular catheter, which information obtained by the nurse is most important to communicate to the health care provider? a. Oral temperature 101.6° F b. Apical pulse 102 beats/min c. Intracranial pressure 15 mm Hg d. Mean arterial pressure 90 mm Hg

ANS: A Infection is a serious consideration with ICP monitoring, especially with intraventricular catheters. The temperature indicates the need for antibiotics or removal of the monitor. The ICP, arterial pressure, and apical pulse are all borderline high but require only ongoing monitoring at this time.

Several patients have been hospitalized for diagnosis of neurologic problems. Which patient will the nurse assess first? a. A patient with a transient ischemic attack (TIA) returning from carotid duplex studies b. A patient with a brain tumor who has just arrived on the unit after a cerebral angiogram c. A patient with a seizure disorder who has just completed an electroencephalogram (EEG) d. A patient prepared for a lumbar puncture whose health care provider is waiting for assistance

ANS: B Because cerebral angiograms require insertion of a catheter into the femoral artery, bleeding is a possible complication. The nurse will need to check the pulse, blood pressure, and the catheter insertion site in the groin as soon as the patient arrives. Carotid duplex studies and EEG are noninvasive. The nurse will need to assist with the lumbar puncture as soon as possible, but monitoring for hemorrhage after cerebral angiogram has a higher priority.

Which nursing action will be most effective in ensuring daily medication compliance for a patient with mild dementia? a. Setting the medications up monthly in a medication box b. Having the patient's family member administer the medication c. Posting reminders to take the medications in the patient's house d. Calling the patient weekly with a reminder to take the medication

ANS: B Because the patient with mild dementia will have difficulty with learning new skills and forgetfulness, the most appropriate nursing action is to have someone else administer the drug. The other nursing actions will not be as effective in ensuring that the patient takes the medications.

A 56-year-old patient in the outpatient clinic is diagnosed with mild cognitive impairment (MCI).Which action will the nurse include in the plan of care? a. Suggest a move into an assisted living facility. b. Schedule the patient for more frequent appointments. c. Ask family members to supervise the patient's daily activities. d. Discuss the preventive use of acetylcholinesterase medications.

ANS: B Ongoing monitoring is recommended for patients with MCI. MCI does not interfere with activities of daily living, acetylcholinesterase drugs are not used for MCI, and an assisted living facility is not indicated for MCI.

A patient who has severe Alzheimer's disease (AD) is being admitted to the hospital for surgery. Which intervention will the nurse include in the plan of care? a. Encourage the patient to discuss events from the past. b. Maintain a consistent daily routine for the patient's care. c. Reorient the patient to the date and time every 2 to 3 hours. d. Provide the patient with current newspapers and magazines.

ANS: B Providing a consistent routine will decrease anxiety and confusion for the patient. Reorientation to time and place will not be helpful to the patient with severe AD, and the patient will not be able to read. The patient with severe AD will probably not be able to remember events from the past.

Which action will help the nurse determine whether a new patient's confusion is caused by dementia or delirium? a. Administer the Mini-Mental Status Exam. b. Use the Confusion Assessment Method tool. c. Determine whether there is a family history of dementia. d. Obtain a list of the medications that the patient usually takes.

ANS: B The Confusion Assessment Method tool has been extensively tested in assessing delirium.

Which action will help the nurse determine whether a new patient's confusion is caused by dementia or delirium? a. Administer the Mini-Mental Status Exam. b. Use the Confusion Assessment Method tool. c. Determine whether there is a family history of dementia. d. Obtain a list of the medications that the patient usually takes.

ANS: B The Confusion Assessment Method tool has been extensively tested in assessing delirium. The other actions will be helpful in determining cognitive function or risk factors for dementia or delirium, but they will not be useful in differentiating between dementia and delirium.

A patient with a head injury opens the eyes to verbal stimulation, curses when stimulated, and does not respond to a verbal command to move but attempts to remove a painful stimulus. The nurse records the patient's Glasgow Coma Scale score as

ANS: B The patient has a score of 3 for eye opening, 3 for best verbal response, and 5 for best motor response.

Which intervention will the nurse include in the plan of care for a patient with moderate dementia who had an appendectomy 2 days ago? a. Provide complete personal hygiene care for the patient. b. Remind the patient frequently about being in the hospital. c. Reposition the patient frequently to avoid skin breakdown. d. Place suction at the bedside to decrease the risk for aspiration.

ANS: B The patient with moderate dementia will have problems with short- and long-term memory and will need reminding about the hospitalization. The other interventions would be used for a patient with severe dementia, who would have difficulty with swallowing, self-care, and immobility.

Which nursing actions could the nurse delegate to a licensed practical/vocational nurse (LPN/LVN) who is part of the team caring for a patient with Alzheimer's disease (select all that apply)? a. Develop a plan to minimize difficult behavior. b. Administer the prescribed memantine (Namenda). c. Remove potential safety hazards from the patient's environment. d. Refer the patient and caregivers to appropriate community resources. e. Help the patient and caregivers choose memory enhancement methods. f. Evaluate the effectiveness of the prescribed enteral feedings on patient nutrition.

ANS: B, C LPN/LVN education and scope of practice includes medication administration and monitoring for environmental safety in stable patients. Planning of interventions such as ways to manage behavior or improve memory, referrals, and evaluation of the effectiveness of interventions require registered nurse (RN)-level education and scope of practice.

Which assessments will the nurse make to monitor a patient's cerebellar function (select all that apply)? a. Test for graphesthesia. b. Observe arm swing with gait. c. Perform the finger-to-nose test. d. Assess heat and cold sensation. e. Measure strength against resistance.

ANS: B, C The cerebellum is responsible for coordination and is assessed by looking at the patient's gait and the finger-to-nose test. The other assessments will be used for other parts of the neurologic assessment.

The spouse of a 67-year-old male patient with early stage Alzheimer's disease (AD) tells the nurse, "I am exhausted from worrying all the time. I don't know what to do." Which actions are best for the nurse to take next (select all that apply)? a. Suggest that a long-term care facility be considered. b. Offer ideas for ways to distract or redirect the patient. c. Teach the spouse about adult day care as a possible respite. d. Suggest that the spouse consult with the physician for antianxiety drugs. e. Ask the spouse what she knows and has considered about dementia care options.

ANS: B, C, E The stress of being a caregiver can be managed with a multicomponent approach. This includes respite care, learning ways to manage challenging behaviors, and further assessment of what the spouse may already have considered for care options. The patient is in the early stages and does not need long-term placement. Antianxiety medications may be appropriate, but other measures should be tried first.

The nurse is administering a mental status examination to a 48-year-old patient who has hypertension. The nurse suspects depression when the patient responds to the nurse's questions with a. "Is that right?" b. "I don't know." c. "Wait, let me think about that." d. "Who are those people over there?"

ANS: B-"I don't know." Answers such as "I don't know" are more typical of depression than dementia. The response "Who are those people over there?" is more typical of the distraction seen in a patient with delirium. The remaining two answers are more typical of a patient with mild to moderate dementia.

The nurse's initial action for a patient with moderate dementia who develops increased restlessness and agitation should be to a. reorient the patient to time, place, and person. b. administer a PRN dose of lorazepam (Ativan). c. assess for factors that might be causing discomfort. d. assign unlicensed assistive personnel (UAP) to stay in the patient's room.

ANS: C Increased motor activity in a patient with dementia is frequently the patient's only way of responding to factors like pain, so the nurse's initial action should be to assess the patient for any precipitating factors. Administration of sedative drugs may be indicated, but this should not be done until assessment for precipitating factors has been completed and any of these factors have been addressed. Reorientation is unlikely to be helpful for the patient with moderate dementia. Assigning UAP to stay with the patient may also be necessary, but any physical changes that may be causing the agitation should be addressed first.

When the nurse applies a painful stimulus to the nail beds of an unconscious patient, the patient responds with internal rotation, adduction, and flexion of the arms. The nurse documents this as a. flexion withdrawal. b. localization of pain. c. decorticate posturing. d. decerebrate posturing.

ANS: C Internal rotation, adduction, and flexion of the arms in an unconscious patient is documented as decorticate posturing. Extension of the arms and legs is decerebrate posturing. Because the flexion is generalized, it does not indicate localization of pain or flexion withdrawal.

A 68-year-old patient is diagnosed with moderate dementia after multiple strokes. During assessment of the patient, the nurse would expect to find a. excessive nighttime sleepiness. b. difficulty eating and swallowing. c. loss of recent and long-term memory. d. fluctuating ability to perform simple tasks.

ANS: C Loss of both recent and long-term memory is characteristic of moderate dementia

A 68-year-old patient is diagnosed with moderate dementia after multiple strokes. During assessment of the patient, the nurse would expect to find a. excessive nighttime sleepiness. b. difficulty eating and swallowing. c. loss of recent and long-term memory. d. fluctuating ability to perform simple tasks.

ANS: C Loss of both recent and long-term memory is characteristic of moderate dementia. Patients with dementia have frequent nighttime awakening. Dementia is progressive, and the patient's ability to perform tasks would not have periods of improvement. Difficulty eating and swallowing is characteristic of severe dementia.

While caring for a patient who has just been admitted with meningococcal meningitis, the RN observes all of the following. Which one requires action by the RN? a. The bedrails at the head and foot of the bed are both elevated. b. The patient receives a regular diet from the dietary department. c. The nursing assistant goes into the patient's room without a mask. d. The lights in the patient's room are turned off and the blinds are shut.

ANS: C Meningococcal meningitis is spread by respiratory secretions, so it is important to maintain respiratory isolation as well as standard precautions. Because the patient may be confused and weak, bedrails should be elevated at both the food and head of the bed. Low light levels in the room decrease pain caused by photophobia. Nutrition is an important aspect of care in a patient with meningitis.

A 71-year-old patient with Alzheimer's disease (AD) who is being admitted to a long-term care facility has had several episodes of wandering away from home. Which action will the nurse include in the plan of care? a. Reorient the patient several times daily. b. Have the family bring in familiar items. c. Place the patient in a room close to the nurses' station. d. Ask the patient why the wandering episodes have occurred.

ANS: C Patients at risk for problems with safety require close supervision. Placing the patient near the nurse's station will allow nursing staff to observe the patient more closely. The use of "why" questions can be frustrating for patients with AD because they are unable to understand clearly or verbalize the reason for wandering behaviors. Because of the patient's short-term memory loss, reorientation will not help prevent wandering behavior. Because the patient had wandering behavior at home, familiar objects will not prevent wandering.

A 71-year-old patient with Alzheimer's disease (AD) who is being admitted to a long-term care facility has had several episodes of wandering away from home. Which action will the nurse include in the plan of care? a. Reorient the patient several times daily. b. Have the family bring in familiar items. c. Place the patient in a room close to the nurses' station. d. Ask the patient why the wandering episodes have occurred.

ANS: C-Place the patient in a room close to the nurses' station. Patients at risk for problems with safety require close supervision. Placing the patient near the nurse's station will allow nursing staff to observe the patient more closely

When administering a mental status examination to a patient with delirium, the nurse should a. wait until the patient is well-rested. b. administer an anxiolytic medication. c. choose a place without distracting stimuli. d. reorient the patient during the examination.

ANS: C-choose a place without distracting stimuli. Because overstimulation by environmental factors can distract the patient from the task of answering the nurse's questions, these stimuli should be avoided. The nurse will not wait to give the examination because action to correct the delirium should occur as soon as possible. Reorienting the patient is not appropriate during the examination. Antianxiety medications may increase the patient's delirium.

After change-of-shift report on the Alzheimer's disease/dementia unit, which patient will the nurse assess first? a. Patient who has not had a bowel movement for 5 days b. Patient who has a stage II pressure ulcer on the coccyx c. Patient who is refusing to take the prescribed medications d. Patient who developed a new cough after eating breakfast

ANS: D A new cough after a meal in a patient with dementia suggests possible aspiration and the patient should be assessed immediately. The other patients also require assessment and intervention, but not as urgently as a patient with possible aspiration or pneumonia.

hemorrhagic stroke

a stroke that results from bleeding into the brain tissue itself (intracerebral or intraparenchymal hemorrhage) or into the subarachnoid space or ventricles (subarachnoid hemorrhage or intraventricular hemorrhage)

The nurse is concerned about a postoperative patient's risk for injury during an episode of delirium. The most appropriate action by the nurse is to a. secure the patient in bed using a soft chest restraint. b. ask the health care provider to order an antipsychotic drug. c. instruct family members to remain with the patient and prevent injury. d. assign unlicensed assistive personnel (UAP) to stay with the patient and offer reorientation.

ANS: D The priority goal is to protect the patient from harm. Having a UAP stay with the patient will ensure the patient's safety. Visits by family members are helpful in reorienting the patient, but families should not be responsible for protecting patients from injury. Antipsychotic medications may be ordered, but only if other measures are not effective because these medications have many side effects. Restraints are not recommended because they can increase the patient's agitation and disorientation.

A 72-year-old female patient is brought to the clinic by the patient's spouse, who reports that she is unable to solve common problems around the house. To obtain information about the patient's current mental status, which question should the nurse ask the patient? a. "Are you sad?" b. "How is your self-image?" c. "Where were you were born?" d. "What did you eat for breakfast?"

ANS: D This question tests the patient's short-term memory, which is decreased in the mild stage of Alzheimer's disease or dementia. Asking the patient about her birthplace tests for remote memory, which is intact in the early stages. Questions about the patient's emotions and self-image are helpful in assessing emotional status, but they are not as helpful in assessing mental state

A 72-year-old female patient is brought to the clinic by the patient's spouse, who reports that she is unable to solve common problems around the house. To obtain information about the patient's current mental status, which question should the nurse ask the patient? a. "Are you sad?" b. "How is your self-image?" c. "Where were you were born?" d. "What did you eat for breakfast?"

ANS: D-"What did you eat for breakfast?" This question tests the patient's short-term memory, which is decreased in the mild stage of Alzheimer's disease or dementia

The nurse is concerned about a postoperative patient's risk for injury during an episode of delirium. The most appropriate action by the nurse is to a. secure the patient in bed using a soft chest restraint. b. ask the health care provider to order an antipsychotic drug. c. instruct family members to remain with the patient and prevent injury. d. assign unlicensed assistive personnel (UAP) to stay with the patient and offer reorientation.

ANS: D-assign unlicensed assistive personnel (UAP) to stay with the patient and offer reorientation.

Pre-test practice questions: The nurse in the long-term care facility cares for a 70-year-old man with severe (late-stage) dementia who is undernourished and has problems chewing and swallowing. What should the nurse include in the plan of care for this patient? A) Turn on the television to provide a distraction during meals. B) Provide thickened fluids and moist foods in bite-size pieces. C) Limit fluid intake during scheduled meals to prevent aspiration. D) Allow the patient to select favorite foods from the menu choices.

B) Provide thickened fluids and moist foods in bite-size pieces. Rationale: If patients with dementia have problems chewing or swallowing, pureed foods, thickened liquids, and nutritional supplements should be provided. Foods that are easy to swallow are moist and should be in bite-size pieces. Distractions at mealtimes, including the television, should be avoided. Fluids should not be limited but offered frequently; fluids should be thickened. Patients with severe (late-stage) dementia have difficulty understanding words and would not have the cognitive ability to select menu choices.

NCLEX review questions: The patient has been diagnosed with the mild cognitive impairment stage of Alzheimer's disease. What nursing interventions should the nurse expect to use with this patient? A) Treat disruptive behavior with antipsychotic drugs. B) Use a calendar and family pictures as memory aids. C) Use a writing board to communicate with the patient. D) Use a wander guard mechanism to keep the patient in the area.

B) Use a calendar and family pictures as memory aids. Rationale: The patient with mild cognitive impairment will have problems with memory, language, or another essential cognitive function that is severe enough to be noticeable to others but does not interfere with activities of daily living. A calendar and family pictures for memory aids will help this patient. This patient should not yet have disruptive behavior or get lost easily. Using a writing board will not help this patient with communication.

Assisting the family to understand what is happening to the patient is an especially important role of the nurse when the patient has a tumor of the a. ventricles b. frontal lobe c. parietal lobe d. occipital lobe

B. Frontal lobe tumors often lead to loss of emotional control, confusion, memory loss, disorientation, and personality changes that are very disturbing and frightening to the family. Physical symptoms, such as blindness, disturbances in sensation and perception, and even seizures, that occur with other tumors are more likely to be understood and accepted by the family

A patient with an intracranial problem does not open his eyes to any stimulus, has no verbal response except moaning and muttering when stimulated, and flexes his arm in response to painful stimuli. The nurse records the patients GCS score as a. 6 b. 7 c. 9 d. 11

B. no opening of eyes = 1 incomprehensible words= 2 flexion withdrawal = 4 Total = 7

A female patient has left-sided hemiplegia following an ischemic stroke that she experienced 4 days earlier. How should the nurse best promote the health of the patient's integumentary system? A. Position the patient on her weak side the majority of the time. B. Alternate the patient's positioning between supine and side-lying. C. Avoid the use of pillows in order to promote independence in positioning. D. Establish a schedule for the message of areas where skin breakdown emerges.

B. Alternate the patient's positioning between supine and side-lying. A position change schedule should be established for stroke patients. An example is side-back-side, with a maximum duration of 2 hours for any position. The patient should be positioned on the weak or paralyzed side for only 30 minutes. Pillows may be used to facilitate positioning. Areas of skin breakdown should never be massaged.

The nurse would expect to find what clinical manifestation in a patient admitted with a left-sided stroke? A. Impulsivity B. Impaired speech C. Left-side neglect D. Short attention span

B. Impaired speech Clinical manifestations of left-sided brain damage include right hemiplegia, impaired speech/language, impaired right/left discrimination, and slow and cautious performance. Impulsivity, left-sided neglect, and short attention span are all manifestations of right-sided brain damage.

The physician orders alteplase (Activase) for a 58-year-old man diagnosed with an acute ischemic stroke. Which nursing action is most appropriate? A. Administer the medication by an IV route at 15 mL/hr for 24 hours. B. Insert two or three large-bore IV catheters before administering the medication. C. If gingival bleeding occurs, discontinue the medication and notify the physician. D. Reduce the medication infusion rate for a systolic blood pressure above 180 mm Hg.

B. Insert two or three large-bore IV catheters before administering the medication. Before giving alteplase, the nurse should start two or three large bore IVs. Bleeding is a major complication with fibrinolytic therapy, and venipunctures should not be attempted after alteplase is administered. Altepase is administered IV with an initial bolus dose followed by an infusion of the remaining medication within the next 60 minutes. Gingival bleeding is a minor complication and may be controlled with pressure or ice packs. Control of blood pressure is critical prior to altepase administration and for the following 24 hours. Before administering altepase, a systolic pressure above 180 mm Hg or diastolic pressure above 110 mm Hg requires aggressive blood pressure treatment to reduce the risk of cerebral hemorrhage.

A diagnosis of a ruptured cerebral aneurysm has been made in a patient with manifestations of a stroke. The nurse anticipates that treatment options that would be evaluated for the patient include a. hyperventilation therapy b. surgical clipping of the aneurysm c. administration of hyperosmotic agents d. administration of thrombolytic therapy

B: Surgical clipping of they aneurysm- Surgical management with clipping of an aneurysm to decrease re bleeding and vasospasm is an option for a stroke cause by rupture of a cerebral aneurysm. Placement of coils into the lumens of the aneurysm by intercentional radiologists is increasing in popularity. Hyperventilation therapy would increase vasodilation and the potential for hemorrhage. Thrombolytic therapy would be absolutely contraindicated, and if a vessel is patent, osmotic diuretics may leak into tissue, pulling fluid out of the vessel and increasing edema.

Vasogenic cerebral edema increases intracranial pressure by A. Shifting fluid in the gray matter B. Altering the endothelial lining of the cerebral capillaries C. Leaking molecules from the intercellular fluid to the capillaries D. Altering the osmotic gradient flow into the intravascular component

Correct answer: b Rationale: Vasogenic cerebral edema occurs mainly in the white matter. It is caused by changes in the endothelial lining of cerebral capillaries.

transient ischemic attack (TIA)

a transient episode of neurologic dysfunction caused by focal brain, spinal cord, or retinal ischemia, but without acute infarction of the brain. Clinical symptoms typically last less than 1 hour

A 54-year old man is recovering from a skull fracture with a subacute subdural hematoma. He has return of motor control and orientation but appears apathetic and has reduced awareness of his environment. When planning discharge or the patient, the nurse explains to the patient and the family that a. continuous improvement in the patient's condition should occur until he has returned to pre trauma status b. the patient's complete recovery may take years, and the family should plan for his long term dependent care c. the patient is likely to have long term emotional and mental changes that may require continued professional help d. role changes in family members will be necessary because the patient will be dependent on his family for care and support

C. Residual mental and emotional changes of brain trauma with personality changes are often the most incapacitating problems following head injury and are common in patients who have been comatose longer than 6 hours. Families must be prepared for changes in the patient's behavior to avoid family-patient friction and maintain family functioning, and professional assistance may be required. There is no indication he will be dependent on others for care, but he likely will not return to pre trauma status

On physical examination of a patient with headache and fever, the nurse would suspect a brain abscess when the patient has a. seizures b. nuchal rigidity c. focal symptoms d. signs of increased ICP

C. The symptoms of brain abscess closely resemble those of meningitis and encephalitis, including fever, headache, and increased ICP, except the patient also usually has some focal symptoms that reflect the local are of the abscess.

The nurse is planning psychosocial support for the patient and family of the patient who suffered a stroke. What factor will most likely have the greatest impact on positive family coping with the situation? A. Specific patient neurologic deficits B. The patient's ability to communicate C. Rehabilitation potential of the patient D. Presence of complications of a stroke

C. Rehabilitation potential of the patient Although a patient's neurologic deficit might initially be severe after a stroke, the ability of the patient to recover is most likely to positively impact the family's coping with the situation. Providing explanations and emotional support beginning in the acute phase through the rehabilitation phase will facilitate coping. Emphasizing successes will offer the most realistic hope for the patient's rehabilitation and helps maintain hope for the patient's future abilities.

Which sensory-perceptual deficit is associated with left-sided stroke (right hemiplegia)? A. Overestimation of physical abilities. B. Difficulty judging position and distance. C. Slow and possibly fearful performance of tasks. D. Impulsivity and impatience at performing tasks.

C. Slow and possibly fearful performance of tasks. Patients with a left-sided stroke (right hemiplegia) commonly are slower in organization and performance of tasks and may have a fearful, anxious response to a stroke. Overconfidence, spatial disorientation, and impulsivity are more commonly associated with a right-sided stroke.

Which intervention should the nurse delegate to the LPN when caring for a patient following an acute stroke? a. assess the patient's neurologic status b. assess the patient's gag reflex before beginning feeding c. administer ordered antihypertensives and platelet inhibitors d. teach the patient's caregivers strategies to minimize unilateral neglect

C: Administer ordered antihypertensives and platelet inhibitors- medication administration is within the scope of practice for an LPN. Assessment and teaching are within the scope of practice for the RN.

A thrombus that develops in a cerebral artery does not always cause a loss of neurologic function because a. the body can dissolve the atherosclerotic plaques as they form b. some tissues of the brain do not require constant blood supply to prevent damage c. circulation through the circle of Willis may provide blood supply to the affected area of the brain d. neurologic deficits occur only when major arteries are occluded by thrombus formation around an atherosclerotic plaque

C: Circulation through the circle of Willis may provide blood supply to the affected area of the brain. The communication between cerebral arteries in the circle of Willing provides a collateral circulation, which may maintain circulation to an area of the brain if its original blood supply is obstructed. ALl areas of the brain require constant blood supply, and atherosclerotic plaques are not readily reversed. Neurologic deficits can result from ischemia cause by many factors.

The incidence of ischemic stroke in patients with TIAs and other risk factors is reduced with administration of a. furosemide (Lasix) b. lovastatin (Mevacor) c. daily low dose aspirin d. nimodipine (Nimotop)

C: Daily low dose aspirin- the administration of antiplatelet agents, such as aspirin, dipyridamole (Persantine), and ticlopdipine (Ticlid), reduces the incidence of stroke in those at risk. Anticoagulants are also used for prevention of embolic strokes but increase the risk for hemorrhage. Diuretics are not indicated for stroke prevention other than for their role in controlling BP, and antilipemic agents have bot been found to have a significant effect on stroke prevention. The calcium channel blocker nimodipine is used in patients with subarachnoid hemorrhage to decrease the effects of vasospasm and minimize tissue damage. P.S. I freaking love you and good luck on the final!!

A patient with a stroke has a right sided hemiplegia. The nurse prepares family members to help control behavior changes seen with this type of stroke by teaching them to a. ignore undesirable behaviors manifested by the patient b. provide directions to the patient verbally in small steps c. distract the patient from inappropriate emotional responses d. supervise all activities before allowing the patient to pursue them independently

C: Distract the patient from inappropriate emotional responses- patients with left-sided brain damage from stroke often experience emotional lability, inappropriate emotional responses, mood swings, and uncontrolled tears or laughter disproportionate or out of context with the situation. The behavior is upsetting and embarrassing to both the patient and the family, and the patient should be distracted to minimize its presence. Patients with right-brain damage often have impulsive, rapid behavior that supervision and direction.

In promoting health maintenance for prevention of strokes, the nurse understands that the highest risk for the most common type of stroke is present in a. African Americans b. women who smoke c.individuals with hypertension and diabetes d. those who are obese with high dietary fat intake

C: Individuals with hypertension and diabetes- The highest risk factors for thrombotic stroke are hypertension and diabetes. African Americans have a higher risk for stroke than do white persons but probably because they have a greater incidence of hypertension. Factors such as obesity, diet high in saturated fats and cholesterol, cigarette smoking, and excessive alcohol use are also risk factors but carry less risk than hypertension.

A carotid endarterectomy is being considered as a treatment for a patient who has had several TIAs. The nurse explains to the patient that this surgery a. is used to restore blood to the brain following an obstruction of a cerebral artery b. involves intracranial surgery to join a superficial extracranial artery to an intracranial artery c. involves removing an atherosclerotic plaque in the carotid artery to prevent an impending stroke d. is sued to open a stenosis in a carotid artery with a balloon and stent to restore cerebral circulation

C: Involves removing an atherosclerotic plaque in the carotid artery to prevent an impending stroke- An endarterectomy is a removal of an atherosclerotic plaque, and plaque in the carotid artery may impair circulation enough to cause a stroke. A carotid endarterectomy is performed to prevent a cerebrovascular accident (CVA), as are most other surgical procedures. An extacranial-intracranial bypass involves cranial surgery to bypass a sclerotic intacranial artery. Percutaneous transluminal angioplasty uses a balloon to compress stenotic areas in the carotid and vertebrobasilar arteries and often includes inserting a stent to hold the artery open.

A patient with right hemisphere stroke has a nursing diagnosis of unilateral neglect related to sensory perceptual deficits. During the patient's rehabilitation, it is important for the nurse to a. avoid positioning the patient on the affected side b. place all objects for care on the patient's unaffected side c. teach the patient to care consciously for the affected side d. protect the affected side from injury with pillows and supports

C: Teach the patient to care consciously for the affected side- unilateral neglect, or neglect syndrome, occurs when the patient with a stroke is unaware of the affected side of the body, which puts the patient at risk for injury. During the acute phase, the affected side is cared for by the nurse with positioning and support, during rehabilitation the patient is taught to care consciously for and attend to the affected side of the body to protect it from injury. Patients may be positioned on the affected side for up to 30 minutes.

The neurologic functions that are affected by a stroke are primarily related to a. the amount of tissue area involved b. the rapidity of onset of symptoms c. the brain area perfused by the affected artery d. the presence or absence of collateral circulation

C: The brain area perfused by the affected artery- clinical manifestation of altered neurologic function differ, depending primarily on the specific cerebral artery involved and the area of the brain that is perfused by the artery. The degree of impairment depends on rapidity of onset, the size of the lesion, and the presence of collateral circulation.

An appropriate food for a patient with a stroke who has mild dysphagia is a. fruit juices b. pureed meat c. scrambled eggs d. fortified milkshakes

C: scrambled eggs- soft foods that provide enough texture, flavor, and bulk to stimulate swallowing should be used for the patient with dysphasia. Thin liquids are difficult to swallow, and patients may not be able to control them in the mouth. Pureed foods are often too bland and to smooth, and milk products should be avoided because they tend to increase the viscosity of mucus and increase salivation.

A patient is suspected of having a brain tumor. The s/s include: memory deficits, visual disturbances, weakness of right upper and lower extremities and personality changes. The nurse recognizes that the tumor is most likely located in the a. Frontal lobe b. Parietal lobe c. Occipital lobe d. Temporal lobe

Correct answer: a Rationale: A unilateral frontal lobe tumor may result in the following signs and symptoms: unilateral hemiplegia, seizures, memory deficit, personality and judgment changes, and visual disturbances. A bilateral frontal lobe tumor may cause symptoms associated with a unilateral frontal lobe tumor and an ataxic gait.

The nurse on clinical unit is assigned to four patients. Which patient should she assess first? a. Patient with a skull fracture whose nose is bleeding b. Older patient with a stroke who is confused and whose daughter is present c. Patient with meningitis who is suddenly agitated and reporting a headache of 10 on a 0-10 scale d. Patient who had a craniotomy for a brain tumor who now 3 days postoperative had had continued vomiting

Correct answer: c Rationale: The patient with meningitis should be seen first; patients with meningitis must be observed closely for manifestations of elevated ICP, which is thought to result from swelling around the dura and increased cerebrospinal fluid (CSF) volume. Sudden change in the level of consciousness or change in behavior along with a sudden severe headache may indicate an acute elevation of ICP. The patient who has undergone cranial surgery should be seen second; although nausea and vomiting are common after cranial surgery, it can result in elevations of ICP. Nausea and vomiting should be treated with antiemetics. The patient with a skull fracture needs to be evaluated for CSF leakage occurring with the nose bleed and should be seen third. Confusion after a stroke may be expected; the patient should have a family member present.

Nursing management of a patient with a brain tumor includes (select all that apply): a. discussing with the patient methods to control inappropriate behavior. b. using diversion techniques to keep the patient stimulated and motivated. c. assisting and supporting the family in understanding any changes in behavior. d. limiting self-care activities until the patient has regained maximum physical functioning. e. planning for seizure precautions and teaching the patient and the caregiver about anti seizure drugs.

Correct answers: c, e Rationale: Nursing interventions should be based on a realistic appraisal of the patient's condition and prognosis after cranial surgery. The nurse should provide support and education to the caregiver and family about the patient's behavioral changes. The nurse should be prepared to manage seizures and teach the caregiver and family about antiseizure medications and how to manage a seizure. An overall goal is to foster the patient's independence for as long as possible and to the highest degree possible. The nurse should decrease stimuli in the patient's environment to prevent increases in intracranial pressure.

Pre-test practice questions: The home care nurse is visiting patients in the community. Which patient is exhibiting an early warning sign of Alzheimer's disease? A) A 65-year-old male does not recognize his family members and close friends. B) A 59-year-old female misplaces her purse and jokes about having memory loss. C) A 79-year-old male is incontinent and not able to perform hygiene independently. D) A 72-year-old female is unable to locate the address where she has lived for 10 years.

D) A 72-year-old female is unable to locate the address where she has lived for 10 years. Rationale: An early warning sign of Alzheimer's disease is disorientation to time and place such as geographic disorientation. Occasionally misplacing items and joking about memory loss are examples of normal forgetfulness. Impaired ability to recognize family and close friends is a clinical manifestation of middle or moderate dementia (or Alzheimer's disease). Incontinence and inability to perform self-care activities are clinical manifestations of severe or late dementia (or Alzheimer's disease).

NCLEX review questions: Which patient may face the greatest risk of developing delirium? A) A patient with fibromyalgia whose chronic pain has recently worsened B) A patient with a fracture who has spent the night in the emergency department C) An older patient whose recent computed tomography (CT) shows brain atrophy D) An older patient who takes multiple medications to treat various health problems

D) An older patient who takes multiple medications to treat various health problems Rationale: Polypharmacy is implicated in many cases of delirium, and this phenomenon is especially common among older adults. Brain atrophy, if associated with cognitive changes, is indicative of dementia. Alterations in sleep and environment, as well as pain, may cause delirium, but this is less of a risk than in an older adult who takes multiple medications.

Metabolic and nutritional needs of the patient with increased ICP are best met with a. enteral feedings that are low in sodium b. the simple glucose available in D5W IV solutions c. a fluid restriction that promotes a moderate dehydration d. balanced, essential nutrition in a form that the patient can tolerate

D. A patient with increased ICP is in a hypermetabolic and hypercatabolic state and needs adequate glucose to maintain fuel for the brain and other nutrients to meet metabolic needs. Malnutrition promotes cerebral edema, and if a patient cannot take oral nutrition, other means of providing nutrition should be used, such as tube feedings or parenteral nutrition. Glucose alone is not adequate to meet nutritional requirements, and 5% dextrose solutions may increase cerebral edema by lowering serum osmolarity. Patients should remain in a normovolemic fluid state with close monitoring of clinical factors such as urine output, fluid intake, serum and urine osmolality, serum electrolytes, and insensible losses.

The nurse suspects the presence of an arterial epidural hematoma in the patient who experiences a. failure to regain consciousness following a head injury b. a rapid deterioration of neurologic function within 24 to 48 hours following a head injury c. nonspecific, nonlocalizing progression of alteration in LOC occurring over weeks or months d. unconsciousness at the time of a head injury with a brief period of consciousness followed by a decrease in LOC

D. An arterial epidural hematoma is the most acute neurologic emergency, and the typical symptoms include unconsciousness at the scene, with a brief lucid interval followed by a decrease in LOC. An acute subdural hematoma manifests signs within 48 hours of an injury; a chronic subdural hematoma develops over weeks or months

A patient is admitted to the hospital with possible bacterial meningitis. During the initial assessment, the nurse questions the patient about a recent history of a. mosquito or tick bites b. chickenpox or measles c. cold sores or fever blisters d. an upper respiratory infection

D. Meningitis is often a result of an upper respiratory infection or middle ear infection, where organisms gain entry to the CNS. Epidemic encephalitis is transmitted by ticks and mosquitoes, and nonepidemic encephalitis may occur as a complication of measles, chickenpox, or mumps. Encephalitis caused by the herpes simplex virus carries a high fatality rate

Skull radiographs and a computed tomography (CT) scan provide evidence of a depressed parietal fracture with a subdural hematoma in a patient admitted to the emergency department following an automobile accident. In planning care for the patient, the nurse anticipates that a. the patient will receive life-support measures until the condition stabilizes b. immediate burr holes will be made to rapidly decompress the intracranial activity c. the patient will be treated conservatively with close monitoring for changes in neurologic condition d. the patient will be taken to surgery for a craniotomy for evacuation of blood and decompression of the cranium

D. When there is a depressed fracture and fractures with loose fragments, a craniotomy is indicated to elevate the depressed bone and remove free fragments. A craniotomy is also indicated in cases of acute subdural and epidural hematomas to remove the blood and control the bleeding. Burr holes may be used in an extreme emergency for rapid decompression, but with a depressed fracture, surgery would be the treatment of choice

The nurse is teaching a senior citizen's group about signs and symptoms of a stroke. Which statement by the nurse would provide accurate information? A. "Take the person to the hospital if a headache lasts for more than 24 hours." B. "Stroke symptoms usually start when the person is awake and physically active." C. "A person with a transient ischemic attack has mild symptoms that will go away." D. "Call 911 immediately if a person develops slurred speech or difficulty speaking."

D. "Call 911 immediately if a person develops slurred speech or difficulty speaking." Medical assistance should be obtained immediately for someone with signs and symptoms of a stroke such as sudden numbness; weakness; paralysis of the face, arm, or leg (especially on one side of the body); sudden confusion; trouble speaking or understanding; slurred speech; sudden trouble seeing in one or both eyes; sudden trouble walking; dizziness; loss of balance or coordination; or a sudden, severe headache with no known cause. A person with signs and symptoms of a transient ischemic attack should seek medical attention immediately because it is unknown if the symptoms will resolve or persist and progress to a stroke. Onset of signs and symptoms of a stroke vary depending on the type. Onset of an ischemic thrombotic stroke usually occurs at rest. Onset of an ischemic embolic stroke is not related to rest or activity, and onset of a hemorrhagic stroke usually occurs with activity.

Of the following patients, the nurse recognizes that the one with the highest risk for stroke is a(n): A. obese 45-year old Native American. B. 35-year-old Asian American woman who smokes. C. 32-year-old white woman taking oral contraceptives. D. 65-year-old African American man with hypertension.

D. 65-year-old African American man with hypertension. Nonmodifiable risk factors for stroke include age (older than 65 years), male gender, ethnicity or race (incidence is highest in African Americans; next highest in Hispanics, Native Americans/Alaska Natives, and Asian Americans; and next highest in white people), and family history of stroke or personal history of a transient ischemic attack or stroke. Modifiable risk factors for stroke include hypertension (most important), heart disease (especially atrial fibrillation), smoking, excessive alcohol consumption (causes hypertension), abdominal obesity, sleep apnea, metabolic syndrome, lack of physical exercise, poor diet (high in saturated fat and low in fruits and vegetables), and drug abuse (especially cocaine). Other risk factors for stroke include a diagnosis of diabetes mellitus, increased serum levels of cholesterol, birth control pills (high levels of progestin and estrogen), history of migraine headaches, inflammatory conditions, hyperhomocystinemia, and sickle cell disease.

The nurse in a primary care provider's office is assessing several patients today. Which patient is most at risk for a stroke? A. A 92-year-old female who takes warfarin (Coumadin) for atrial fibrillation. B. A 28-year-old male who uses marijuana after chemotherapy to control nausea. C. A 42-year-old female who takes oral contraceptives and has migraine headaches. D. A 72-year-old male who has hypertension and diabetes mellitus and smokes tobacco.

D. A 72-year-old male who has hypertension and diabetes mellitus and smokes tobacco. Stroke risk increases after 65 years of age. Strokes are more common in men. Hypertension is the single most important modifiable risk factor for stroke. Diabetes mellitus is a significant stroke risk factor; and smoking nearly doubles the risk of a stroke. Other risk factors include drug abuse (especially cocaine), high-dose oral contraception use, migraine headaches, and untreated heart disease such as atrial fibrillation.

A 74-year-old man who has right-sided extremity paralysis related to a thrombotic stroke develops constipation. Which action should the nurse take first? A. Assist the patient to the bathroom every 2 hours. B. Provide incontinence briefs to wear during the day. C. Administer a bisocodyl (Dulcolax) rectal suppository every day. D. Arrange for several servings per day of cooked fruits and vegetables.

D. Arrange for several servings per day of cooked fruits and vegetables. Patients after a stroke frequently have constipation. Dietary management includes the following: fluid intake of 2500 to 3000 mL daily, prune juice (120 mL) or stewed prunes daily, cooked fruit three times daily, cooked vegetables three times daily, and whole-grain cereal or bread three to five times daily. Patients with urinary incontinence should be assisted to the bathroom every 2 hours when appropriate. Suppositories may be ordered for short-term management if the patient does not respond to increased fluid and fiber. Incontinence briefs are indicated as a short-term intervention for urinary incontinence.

The female patient has been brought to the ED with a sudden onset of a severe headache that is different from any other headache she has had previously. When considering the possibility of a stroke, which type of stroke should the nurse know is most likely occurring? A. TIA B. Embolic stroke C. Thrombotic stroke D. Subarachnoid hemorrhage

D. Subarachnoid hemorrhage Headache is common in a patient who has a subarachnoid hemorrhage or an intracerebral hemorrhage. A TIA is a transient loss of neurologic function usually without a headache. A headache may occur with an ischemic embolic stroke, but severe neurologic deficits are the initial symptoms. The ischemic thrombotic stroke manifestations progress in the first 72 hours as infarction and cerebral edema increase.

Which intervention is most appropriate when communicating with a patient suffering from aphasia following a stroke? A. Present several thoughts at once so that the patient can connect the ideas. B. Ask open-ended questions to provide the patient the opportunity to speak. C. Finish the patient's sentences to minimize frustration associated with slow speech. D. Use simple, short sentences accompanied by visual cues to enhance comprehension.

D. Use simple, short sentences accompanied by visual cues to enhance comprehension. When communicating with a patient with aphasia, the nurse should present one thought or idea at a time. Ask questions that can be answered with a "yes," "no," or simple word. Use visual cues and allow time for the individual to comprehend and respond to conversation.

A nursing intervention is indicated for the patient with hemiplegia is a. the use of a footboard to prevent plantar flexion b. immobilization of the affected arm against the chest with a sling c. positioning the patient in bed with each joint lower than the joint proximal to it d. having the patient perform passive ROM of the affected limb with the unaffected limb

D: Having the patient perform passive ROM of the affected limb with the unaffected limb- active ROM should be initiated on the unaffected side as soon as possible, and passive ROM of the affected side should be started on the first day. Having the patient actively exercise the unaffected side provides the patient with active and passive ROM as needed. Use of footboards is controversial because they stimulate plantar flexion. The unaffected arm should be supported, but immobilization may precipitate a painful shoulder-hand syndrome. The patient should be positioned with each joint higher than the joint proximal to it to prevent dependent edema.

The nurse can assist the patient and the family in coping with the long term effects of a stroke by a. informing family members that the patient will need assistance with almost all ADLs b. explaining that the patient's prestroke behavior will return as improvement progresses c. encouraging the patient and family members to seek assistance from family therapy or stroke support groups d. helping the patient and family understand the significance of residual stroke damage to promote problem solving and planning

D: Helping the patient and family understand the significance of residual stroke damage to promote problem solving and planning- the patient and family need accurate and complete information about the effects of the stroke to problem solve and make plans for chronic care of the patient. It is uncommon for patients with major strokes to return completely to pre stroke function, behaviors, and role, and both the patient and family will mourn these losses. The patient's specific needs for care must be identified, and rehabilitation efforts should be continued at home. Family therapy and support groups may be helpful for some patients and families.

A patient's wife asks the nurse why her husband did not receive the clot busting medication (tPA) she has been reading about. Her husband is diagnosed with a hemorrhagic stroke. What should the nurse respond? a. He didn't arrive within the time frame for that therapy b. Not every is eligible for this drug. Has he had surgery lately? c. You should discuss the treatment of your husband with your doctor d. The medication you are talking about dissolves clots and could cause more bleeding in your husband's head

D: The medication you are talking about dissolves clots and could cause more bleeding in your husband's head- tPA dissolves clots and increases the risk for bleeding. It is not used with hemorrhagic strokes. If the patient had a thrombotic/embolic stroke the time frame would be important as well as a history of surgery. The nurse should answer the question as accurately as possible and then encourage the individual to talk with the primary care physician if he or she has further questions.

A patient comes to the emergency department immediately after experiencing numbness of the face and an inability to speak, but while the patient awaits examination, the symptoms disappear and the patient request discharge. The nurse stresses that it is important for the patient to be evaluated primarily because a. the patient has probably experienced an asymptomatic lacunar stroke b. the symptoms are likely to return and progress to worsening neurologic deficit in the next 24 hours c. neurologic deficits that are transient occur most often as a result of small hemorrhages that clot off d. the patient has probably experienced a transient ischemic attack (TIA), which is a sign of progressive cerebral vascular disease

D: The patient has probably experienced a transient ischemic attack (TIA), which is a sign of progressive cerebral vascular disease- A TIA is a temporary focal loss of neurologic function caused by ischemia of an area of the brain, usually lasting only about 3 hours. TIAs may be due to microemboli from heart disease or carotid or cerebral thrombi and are a warning of progressive disease. Evaluation is necessary to determine the cause of the neurologic deficit and provide prophylactic treatment if possible.

13. When assessing a patient with Alzheimer's disease (AD) who is being admitted to a long-term care facility, the nurse learns that the patient has had several episodes of wandering away from home. Which nursing action will the nurse include in the plan of care? a. Place the patient in a room close to the nurses' station. b. Ask the patient why the wandering episodes have occurred. c. Have the family bring in familiar items from the patient's home. d. Reorient the patient to the new living situation several times daily.

a. Place the patient in a room close to the nurses' station. Patients at risk for problems with safety require close supervision. Placing the patient near the nurse's station will allow nursing staff to observe the patient more closely. The use of "why" questions is frustrating for patients with AD because they are unable to understand clearly or verbalize the reason for wandering behaviors. Because of the patient's short-term memory loss, reorientation will not help prevent wandering behavior. Because the patient had wandering behavior at home, familiar objects will not prevent wandering.

1. A patient who is hospitalized with pneumonia is disoriented and confused 2 days after admission. Which information obtained by the nurse about the patient indicates that the patient is experiencing delirium rather than dementia? a. The patient was oriented and alert when admitted. b. The patient's speech is fragmented and incoherent. c. The patient is disoriented to place and time but oriented to person. d. The patient has a history of increasing confusion over several years.

a. The patient was oriented and alert when admitted. The onset of delirium occurs acutely. The degree of disorientation does not differentiate between delirium and dementia. Increasing confusion for several years is consistent with dementia. Fragmented and incoherent speech may occur with either delirium or dementia.

Bridge to NCLEX questions: A major goal of treatment for the patient with AD is to a. maintain patient safety. b. maintain or increase body weight. c. return to a higher level of self-care. d. enhance functional ability over time.

a. maintain patient safety. Rationale: The overall management goals are that the patient with AD will (1) maintain functional ability for as long as possible, (2) be maintained in a safe environment with a minimum of injuries, (3) have personal care needs met, and (4) have dignity maintained. The nurse should place emphasis on patient safety while planning and providing nursing care.

aphasia

an abnormal neurologic condition in which language function is disordered or absent because of an injury to certain areas of the cerebral cortex

For a patient who is suspected of having a stroke, one of the most important pieces of information that the nurse can obtain is a. time of the patient's last meal. b. time at which stroke symptoms first appeared. c. patient's hypertension history and management. d. family history of stroke and other cardiovascular diseases.

b Rationale: During initial evaluation, the most important point in the patient's history is the time since onset of stroke symptoms. If the stroke is ischemic, recombinant tissue plasminogen activator (tPA) must be administered within 3 to 4.5 hours of the onset of clinical signs of ischemic stroke; tPA reestablishes blood flow through a blocked artery and prevents brain cell death in patients with acute onset of ischemic stroke.

Bridge to NCLEX questions: Which statement(s) accurately describe(s) mild cognitive impairment (select all that apply)? a. Always progresses to AD b. Caused by variety of factors and may progress to AD c. Should be aggressively treated with acetylcholinesterase drugs d. Caused by vascular infarcts that, if treated, will delay progression to AD e. Patient is usually not aware that there is a problem with his or her memory

b. Caused by variety of factors and may progress to AD Rationale: Although some individuals with mild cognitive impairment (MCI) revert to normal cognitive function or do not go on to develop Alzheimer's disease (AD), those with MCI are at high risk for AD. No drugs have been approved for the treatment of MCI. A person with MCI is often aware of a significant change in memory.

The family caregiver for a patient with AD expresses an inability to make decisions, concentrate, or sleep. The nurse determines what about the caregiver? a. The caregiver is also developing signs of AD. b. The caregiver is manifesting symptoms of caregiver role strain. c. The caregiver needs a period of respite from care of the patient. d. The caregiver should ask other family members to participate in the patient's care.

b. Family caregiver role strain is characterized by such symptoms of stress as the inability to sleep, make decisions, or concentrate. It is frequently seen in family members who are responsible for the care of the patient with AD. Assessment of the caregiver may reveal a need for assistance to increase coping skills, effectively use community resources, or maintain social relationships. Eventually the demands on a caregiver exceed the resources and the person with AD may be placed in an institutional setting.

12. Which intervention will the nurse include in the plan of care for a patient who has late-stage Alzheimer's disease (AD)? a. Encourage the patient to discuss events from the past. b. Maintain a consistent daily routine for the patient's care. c. Reorient the patient to the date and time every 2 to 3 hours. d. Provide the patient with current newspapers and magazines.

b. Maintain a consistent daily routine for the patient's care. Providing a consistent routine will decrease anxiety and confusion for the patient. In late-stage AD, the patient will not remember events from the past. Reorientation to time and place will not be helpful to the patient with late-stage AD, and the patient will not be able to read.

9. A 62-year-old patient is brought to the clinic by a family member who is concerned about the patient's inability to solve common problems. To obtain information about the patient's current mental status, which question should the nurse ask the patient? a. "Where were you were born?" b. "Do you have any feelings of sadness?" c. "What did you have for breakfast?" d. "How positive is your self-image?"

c. "What did you have for breakfast?" This question tests the patient's recent memory, which is decreased early in Alzheimer's disease (AD) or dementia. Asking the patient about birthplace tests for remote memory, which is intact in the early stages. Questions about the patient's emotions and self-image are helpful in assessing emotional status, but they are not as helpful in assessing mental state.

1. The spouse of a male patient with early stage Alzheimer's disease (AD) tells the nurse, "I am just exhausted from the constant worry. I don't know what to do." Which action is best for the nurse to take next (select all that apply)? a. Suggest that a long-term care facility be considered. b. Offer ideas for ways to distract or redirect the patient. c. Suggest that the spouse consult with the physician for antianxiety drugs. d. Educate the spouse about the availability of adult day care as a respite. e. Ask the spouse what she knows and has considered about dementia care options.

b. Offer ideas for ways to distract or redirect the patient. d. Educate the spouse about the availability of adult day care as a respite. e. Ask the spouse what she knows and has considered about dementia care options. The stress of being a caregiver can be managed with a multicomponent approach. This includes respite care, learning ways to manage challenging behaviors, and further assessment of what the spouse may already have considered. The patient is in the early stages and does not need long-term placement. Antianxiety medications may be appropriate but other measures should be tried first.

2. When developing a plan of care for a hospitalized patient with moderate dementia, which intervention will the nurse include? a. Provide complete personal hygiene care for the patient. b. Remind the patient frequently about being in the hospital. c. Reposition the patient frequently to avoid skin breakdown. d. Place suction at the bedside to decrease the risk for aspiration.

b. Remind the patient frequently about being in the hospital. The patient with moderate dementia will have problems with short- and long-term memory and will need reminding about the hospitalization. The other interventions would be used for a patient with severe dementia, who would have difficulty with swallowing, self-care, and immobility.

5. Which action will the nurse in the outpatient clinic include in the plan of care for a patient with mild cognitive impairment (MCI)? a. Suggest a move into an assisted living facility. b. Schedule the patient for more frequent appointments. c. Ask family members to supervise the patient's daily activities. d. Discuss the preventive use of acetylcholinesterase medications.

b. Schedule the patient for more frequent appointments. Ongoing monitoring is recommended for patients with MCI. MCI does not interfere with activities of daily living, acetylcholinesterase drugs are not used for MCI, and an assisted living facility is not indicated for MCI.

Bridge to NCLEX questions: Creutzfeldt-Jakob disease is characterized by a. remissions and exacerbations over many years. b. memory impairment, muscle jerks, and blindness. c. parkinsonian symptoms, including muscle rigidity and tremors at rest. d. increased intracranial pressure secondary to decreased CSF drainage.

b. memory impairment, muscle jerks, and blindness. Rationale: Creutzfeldt-Jakob disease (CJD) is a fatal brain disorder caused by a prion protein. The earliest symptom of the disease may be memory impairment and behavioral changes. The disease progresses rapidly, with mental deterioration, involuntary movements (i.e., muscle jerks), weakness in the limbs, blindness, and eventually coma

Bridge to NCLEX questions: The early stage of AD is characterized by a. no noticeable change in behavior. b. memory problems and mild confusion. c. increased time spent sleeping or in bed. d. incontinence, agitation, and wandering behavior.

b. memory problems and mild confusion. Rationale: An initial sign of AD is a subtle deterioration in memory.

The nurse explains to the patient with a stroke who is scheduled for angiography that this test is used to determine the a. presence of increased ICP. b. site and size of the infarction. c. patency of the cerebral blood vessels. d. presence of blood in the cerebrospinal fluid.

c Rationale: Angiography provides visualization of cerebral blood vessels and can help estimate perfusion and detect filling defects in the cerebral arteries.

A patient with right-sided hemiplegia and aphasia resulting from a stroke most likely has involvement of the a. brainstem. b. vertebral artery. c. left middle cerebral artery. d. right middle cerebral artery.

c Rationale: If the middle cerebral artery is involved in a stroke, the expected clinical manifestations include aphasia, motor and sensory deficit, and hemianopsia on the dominant side and include neglect, motor and sensory deficit, and hemianopsia on the nondominant side.

A patient experiencing TIAs is scheduled for a carotid endarterectomy. The nurse explains that this procedure is done to a. decrease cerebral edema. b. reduce the brain damage that occurs during a stroke in evolution. c. prevent a stroke by removing atherosclerotic plaques blocking cerebral blood flow. d. provide a circulatory bypass around thrombotic plaques obstructing cranial circulation.

c Rationale: In a carotid endarterectomy, the atheromatous lesion is removed from the carotid artery to improve blood flow.

Bladder training in a male patient who has urinary incontinence after a stroke includes a. limiting fluid intake. b. keeping a urinal in place at all times. c. assisting the patient to stand to void. d. catheterizing the patient every 4 hours.

c Rationale: In the acute stage of stroke, the primary urinary problem is poor bladder control and incontinence. Nurses should promote normal bladder function and avoid the use of indwelling catheters. A bladder retraining program consists of (1) adequate fluid intake, with most fluids administered between 7:00 am and 7:00 pm; (2) scheduled toileting every 2 hours with the use of a bedpan, commode, or bathroom; and (3) noting signs of restlessness, which may indicate the need for urination. Intermittent catheterization may be used for urinary retention (not urinary incontinence). During the rehabilitation phase after a stroke, nursing interventions focused on urinary continence include (1) assessment for bladder distention by palpation; (2) offering the bedpan, urinal, commode, or toilet every 2 hours during waking hours and every 3 to 4 hours at night; (3) using a direct command to help the patient focus on the need to urinate; (4) assistance with clothing and mobility; (5) scheduling most fluid intake between 7:00 am and 7:00 pm; and (6) encouraging the usual position for urinating (i.e., standing for men and sitting for women).

The factor related to cerebral blood flow that most often determines the extent of cerebral damage from a stroke is the a. amount of cardiac output. b. oxygen content of the blood. c. degree of collateral circulation. d. level of carbon dioxide in the blood.

c Rationale: The extent of the stroke depends on the rapidity of onset, size of the lesion, and presence of collateral circulation.

The optic nerve is tested by

having the patient read a Snellen chart or a newspaper.

Of the following patients, the nurse recognizes that the one with the highest risk for a stroke is a(n) a. obese 45-year-old Native American. b. 35-year-old Asian American woman who smokes. c. 32-year-old white woman taking oral contraceptives. d. 65-year-old African American man with hypertension.

d Rationale: Nonmodifiable risk factors for stroke include age (older than 65 years), male gender, ethnicity or race (incidence is highest in African Americans; next highest in Hispanics, Native Americans/Alaska Natives, and Asian Americans; and next highest in white people), and family history of stroke or personal history of a transient ischemic attack or stroke. Modifiable risk factors for stroke include hypertension (most important), heart disease (especially atrial fibrillation), smoking, excessive alcohol consumption (causes hypertension), abdominal obesity, sleep apnea, metabolic syndrome, lack of physical exercise, poor diet (high in saturated fat and low in fruits and vegetables), and drug abuse (especially cocaine). Other risk factors for stroke include a diagnosis of diabetes mellitus, increased serum levels of cholesterol, birth control pills (high levels of progestin and estrogen), history of migraine headaches, inflammatory conditions, hyperhomocystinemia, and sickle cell disease.

Bridge to NCLEX questions: Which patient is most at risk for developing delirium? a. A 50-year-old woman with cholecystitis b. A 19-year-old man with a fractured femur c. A 42-year-old woman having an elective hysterectomy d. A 78-year-old man admitted to the medical unit with complications related to heart failure

d. A 78-year-old man admitted to the medical unit with complications related to heart failure Rationale: Risk factors that can precipitate delirium include age of 65 years or older, male gender, and severe acute illness (e.g., heart failure). The 78-year-old man has the most risk factors for delirium

8. To determine whether a new patient's confusion is caused by dementia or delirium, which action should the nurse take? a. Assess the patient using the Mini-Mental Status Exam. b. Obtain a list of the medications that the patient usually takes. c. Determine whether there is positive family history of dementia. d. Use the Confusion Assessment Method tool to assess the patient.

d. Use the Confusion Assessment Method tool to assess the patient. The Confusion Assessment Method tool has been extensively tested in assessing delirium. The other actions will be helpful in determining cognitive function or risk factors for dementia or delirium, but they will not be useful in differentiating between dementia and delirium.

4. To protect a patient from injury during an episode of delirium, the most appropriate action by the nurse is to a. secure the patient in bed using a soft chest restraint. b. ask the health care provider about ordering an antipsychotic drug. c. instruct family members to remain with the patient and prevent injury. d. assign a nursing assistant to stay with the patient and offer frequent reorientation.

d. assign a nursing assistant to stay with the patient and offer frequent reorientation. The priority goal is to protect the patient from harm, and a staff member will be most experienced in providing safe care. Visits by family members are helpful in reorienting the patient, but families should not be responsible for protecting patients from injury. Antipsychotic medications may be ordered, but only if other measures are not effective because these medications have multiple side effects. Restraints are sometimes used but tend to increase agitation and disorientation.

3. When administering a mental status examination to a patient with delirium, the nurse should a. medicate the patient first to reduce any anxiety. b. give the examination when the patient is well-rested. c. reorient the patient as needed during the examination. d. choose a place without distracting environmental stimuli.

d. choose a place without distracting environmental stimuli. Because overstimulation by environmental factors can distract the patient from the task of answering the nurse's questions, these stimuli should be avoided. The nurse will not wait to give the examination because action to correct the delirium should occur as soon as possible. Reorienting the patient is not appropriate during the examination. Antianxiety medications may increase the patient's delirium.

7. A 72-year-old patient is diagnosed with moderate dementia as a result of multiple strokes. During assessment of the patient, the nurse would expect to find a. excessive nighttime sleepiness. b. difficulty eating and swallowing. c. variable ability to perform simple tasks. d. loss of both recent and long-term memory.

d. loss of both recent and long-term memory. Loss of both recent and long-term memory is characteristic of moderate dementia. Patients with dementia have frequent nighttime awakening. Dementia is progressive, and the patient's ability to perform tasks would not have periods of improvement. Difficulty eating and swallowing is characteristic of severe dementia.

Bridge to NCLEX questions: The clinical diagnosis of dementia is based on a. CT or MRS. b. brain biopsy. c. electroencephalogram. d. patient history and cognitive assessment.

d. patient history and cognitive assessment. Rationale: The diagnosis of dementia depends on determining the cause. A thorough physical examination is performed to rule out other potential medical conditions. Cognitive testing (e.g., Mini-Mental State Examination) is focused on evaluating memory, ability to calculate, language, visual-spatial skills, and degree of alertness. Diagnosis of dementia related to vascular causes is based on the presence of cognitive loss, the presence of vascular brain lesions demonstrated by neuroimaging techniques, and the exclusion of other causes of dementia. Structural neuroimaging with computed tomography (CT) or magnetic resonance imaging (MRI) is used in the evaluation of patients with dementia. A psychologic evaluation is also indicated to determine the presence of depression.

Bridge to NCLEX questions: Dementia is defined as a a. syndrome that results only in memory loss. b. disease associated with abrupt changes in behavior. c. disease that is always due to reduced blood flow to the brain. d. syndrome characterized by cognitive dysfunction and loss of memory.

d. syndrome characterized by cognitive dysfunction and loss of memory. Rationale: Dementia is a syndrome characterized by dysfunction in or loss of memory, orientation, attention, language, judgment, and reasoning. Personality changes and behavioral problems such as agitation, delusions, and hallucinations may result.

stroke

death of brain cells that occurs when there is ischemia (inadequate blood flow) to a part of the brain or hemorrhage into the brain

dysphasia

difficulty related to the comprehension or use of language

EMG is used to evaluate

electrical innervation to skeletal muscle. De-mylinaion


Kaugnay na mga set ng pag-aaral

ISYS 316 Advanced Java Programming Chapter 32 Java Database Programming

View Set

Principles of Marketing Chapter 11 Quiz

View Set